In modern day combat, what would warrant singing during battle?










33












$begingroup$


Cicad the singer is having a downtime in his career.



Over the millennia, Cicad sang anywhere from royal courts to farmers' cottages and have been rewarded with many, many hand-claps.
But to keep going, indeed to preserve his immortality, he must sing - in battle.



When leading a charge to meet the enemy, Cicad would raise his mighty voice and sword in song, inspire his troops - and his superpower. Yes, the secret behind his long life is twofold:



The first: Singing in battle makes him as strong as three men, and impervious to attack. Even a ballista bolt couldn't scratch him, so long as he's singing with at least two of his men within earshot, fighting against at least two of his enemies.



The second: Singing in battle at least once every couple of years keeps him from aging, thereby letting him live for millennia.



  • At least four men must be fighting, not including himself.

  • The intent to kill the other guys is what constitutes a fight, so they can use long range weapons, even cannons or missiles. However, the field must be such that his guys will have at least caught a glimpse of some of the enemy, at a point in time that's close to the time of the attack (firing a missile at people so far away that you can't ever see them doesn't answer the requirement of "a fight").

  • For the same reason, the fight can't be a mock fight (e.g. practice or a fun bar fight). An intent to kill is required.

  • Earshot means that at least two of his troops must be able to hear his song.

  • Cicad doesn't particularly like to fight, but he's good at it and the troops love him for both the songs and, well, the fact that his side tends to win. So every two years, at least one fight.

This was all well and good in the old days, when a good charge with melee weapons constituted a good fight.



Nowadays, however... There are plenty of fights to be had but not the kind where singing is appreciated. Indeed, as recently as the 19th century Cicad has faced frowns when singing while the battle rages.



In this day and age, where concentration and stealth is a prime concern in battle, he was threatened with removal from duty for singing at the wrong times. This almost made him come forth and tell of his abilities, but he absolutely does not want to be probed and bisected by scientists who'll want to check what makes his superpower tick.



So the question is:
What kind of situations in modern day combat warrant loud singing by at least one particular combatant? The bigger the advantage one side gets by the singing, the better the answer."



  • For the sake of the question, lets assume that other noises (e.g gun/cannon fire, screams of the dying) are not getting in the way of Cicad's singing.

Edit 1:
I really should have clarified this - I want Cicad to be running in the field and getting smacked with fire and brimstone (bullets and heavy ordinance) from time to time. So, infantry. He can be in a tank for a bit of the fight but the point of his imperiousness to harm is to get an unhinged singer into the thick of it. I'll worry about people not noticing his invincibility later...



Edit 2: For those still complaining about other noises, as I wrote - hand (ear?) wave it. Part of the superpower is that the singing subdues noises that are undesired by the audience (which can be a cool advantage by itself, just not so relevant to this question).










share|improve this question











$endgroup$







  • 1




    $begingroup$
    Presumably he's an infantryman? (Because if he's Artillery, Armor, the Corps of Engineers or Service and Support Branches he won't be seeing much.)
    $endgroup$
    – RonJohn
    Aug 26 '18 at 17:47






  • 2




    $begingroup$
    It all depends on who's the leader of the club that's made for you and me.
    $endgroup$
    – Mazura
    Aug 26 '18 at 20:51










  • $begingroup$
    He should turn himself into an Amazee Dayzee. Those things‘ music is super annoying.
    $endgroup$
    – DonielF
    Aug 26 '18 at 23:59










  • $begingroup$
    The question could be broadened to ask in what situation in combat in any era it made sense for a warrior to sing. That uses oxygen and brain power which could better be employed in fighting. Musicians (particularly trumpeters and drummers) have at times had a rôle in boosting morale or communicating orders, but they didn't play with one hand and wield a weapon in the other.
    $endgroup$
    – Peter Taylor
    Aug 27 '18 at 7:47






  • 4




    $begingroup$
    This is a wonderful, wacky question and I love it. "I've been working on the..." BANG! "railroad! All the live long day!" WHACK! "I've been working..." GRUNT! GURGLE! RATATATAT! "working on the raaaailroadddd!" It's like the Joker joined the Marines.
    $endgroup$
    – JBH
    Aug 27 '18 at 19:22















33












$begingroup$


Cicad the singer is having a downtime in his career.



Over the millennia, Cicad sang anywhere from royal courts to farmers' cottages and have been rewarded with many, many hand-claps.
But to keep going, indeed to preserve his immortality, he must sing - in battle.



When leading a charge to meet the enemy, Cicad would raise his mighty voice and sword in song, inspire his troops - and his superpower. Yes, the secret behind his long life is twofold:



The first: Singing in battle makes him as strong as three men, and impervious to attack. Even a ballista bolt couldn't scratch him, so long as he's singing with at least two of his men within earshot, fighting against at least two of his enemies.



The second: Singing in battle at least once every couple of years keeps him from aging, thereby letting him live for millennia.



  • At least four men must be fighting, not including himself.

  • The intent to kill the other guys is what constitutes a fight, so they can use long range weapons, even cannons or missiles. However, the field must be such that his guys will have at least caught a glimpse of some of the enemy, at a point in time that's close to the time of the attack (firing a missile at people so far away that you can't ever see them doesn't answer the requirement of "a fight").

  • For the same reason, the fight can't be a mock fight (e.g. practice or a fun bar fight). An intent to kill is required.

  • Earshot means that at least two of his troops must be able to hear his song.

  • Cicad doesn't particularly like to fight, but he's good at it and the troops love him for both the songs and, well, the fact that his side tends to win. So every two years, at least one fight.

This was all well and good in the old days, when a good charge with melee weapons constituted a good fight.



Nowadays, however... There are plenty of fights to be had but not the kind where singing is appreciated. Indeed, as recently as the 19th century Cicad has faced frowns when singing while the battle rages.



In this day and age, where concentration and stealth is a prime concern in battle, he was threatened with removal from duty for singing at the wrong times. This almost made him come forth and tell of his abilities, but he absolutely does not want to be probed and bisected by scientists who'll want to check what makes his superpower tick.



So the question is:
What kind of situations in modern day combat warrant loud singing by at least one particular combatant? The bigger the advantage one side gets by the singing, the better the answer."



  • For the sake of the question, lets assume that other noises (e.g gun/cannon fire, screams of the dying) are not getting in the way of Cicad's singing.

Edit 1:
I really should have clarified this - I want Cicad to be running in the field and getting smacked with fire and brimstone (bullets and heavy ordinance) from time to time. So, infantry. He can be in a tank for a bit of the fight but the point of his imperiousness to harm is to get an unhinged singer into the thick of it. I'll worry about people not noticing his invincibility later...



Edit 2: For those still complaining about other noises, as I wrote - hand (ear?) wave it. Part of the superpower is that the singing subdues noises that are undesired by the audience (which can be a cool advantage by itself, just not so relevant to this question).










share|improve this question











$endgroup$







  • 1




    $begingroup$
    Presumably he's an infantryman? (Because if he's Artillery, Armor, the Corps of Engineers or Service and Support Branches he won't be seeing much.)
    $endgroup$
    – RonJohn
    Aug 26 '18 at 17:47






  • 2




    $begingroup$
    It all depends on who's the leader of the club that's made for you and me.
    $endgroup$
    – Mazura
    Aug 26 '18 at 20:51










  • $begingroup$
    He should turn himself into an Amazee Dayzee. Those things‘ music is super annoying.
    $endgroup$
    – DonielF
    Aug 26 '18 at 23:59










  • $begingroup$
    The question could be broadened to ask in what situation in combat in any era it made sense for a warrior to sing. That uses oxygen and brain power which could better be employed in fighting. Musicians (particularly trumpeters and drummers) have at times had a rôle in boosting morale or communicating orders, but they didn't play with one hand and wield a weapon in the other.
    $endgroup$
    – Peter Taylor
    Aug 27 '18 at 7:47






  • 4




    $begingroup$
    This is a wonderful, wacky question and I love it. "I've been working on the..." BANG! "railroad! All the live long day!" WHACK! "I've been working..." GRUNT! GURGLE! RATATATAT! "working on the raaaailroadddd!" It's like the Joker joined the Marines.
    $endgroup$
    – JBH
    Aug 27 '18 at 19:22













33












33








33





$begingroup$


Cicad the singer is having a downtime in his career.



Over the millennia, Cicad sang anywhere from royal courts to farmers' cottages and have been rewarded with many, many hand-claps.
But to keep going, indeed to preserve his immortality, he must sing - in battle.



When leading a charge to meet the enemy, Cicad would raise his mighty voice and sword in song, inspire his troops - and his superpower. Yes, the secret behind his long life is twofold:



The first: Singing in battle makes him as strong as three men, and impervious to attack. Even a ballista bolt couldn't scratch him, so long as he's singing with at least two of his men within earshot, fighting against at least two of his enemies.



The second: Singing in battle at least once every couple of years keeps him from aging, thereby letting him live for millennia.



  • At least four men must be fighting, not including himself.

  • The intent to kill the other guys is what constitutes a fight, so they can use long range weapons, even cannons or missiles. However, the field must be such that his guys will have at least caught a glimpse of some of the enemy, at a point in time that's close to the time of the attack (firing a missile at people so far away that you can't ever see them doesn't answer the requirement of "a fight").

  • For the same reason, the fight can't be a mock fight (e.g. practice or a fun bar fight). An intent to kill is required.

  • Earshot means that at least two of his troops must be able to hear his song.

  • Cicad doesn't particularly like to fight, but he's good at it and the troops love him for both the songs and, well, the fact that his side tends to win. So every two years, at least one fight.

This was all well and good in the old days, when a good charge with melee weapons constituted a good fight.



Nowadays, however... There are plenty of fights to be had but not the kind where singing is appreciated. Indeed, as recently as the 19th century Cicad has faced frowns when singing while the battle rages.



In this day and age, where concentration and stealth is a prime concern in battle, he was threatened with removal from duty for singing at the wrong times. This almost made him come forth and tell of his abilities, but he absolutely does not want to be probed and bisected by scientists who'll want to check what makes his superpower tick.



So the question is:
What kind of situations in modern day combat warrant loud singing by at least one particular combatant? The bigger the advantage one side gets by the singing, the better the answer."



  • For the sake of the question, lets assume that other noises (e.g gun/cannon fire, screams of the dying) are not getting in the way of Cicad's singing.

Edit 1:
I really should have clarified this - I want Cicad to be running in the field and getting smacked with fire and brimstone (bullets and heavy ordinance) from time to time. So, infantry. He can be in a tank for a bit of the fight but the point of his imperiousness to harm is to get an unhinged singer into the thick of it. I'll worry about people not noticing his invincibility later...



Edit 2: For those still complaining about other noises, as I wrote - hand (ear?) wave it. Part of the superpower is that the singing subdues noises that are undesired by the audience (which can be a cool advantage by itself, just not so relevant to this question).










share|improve this question











$endgroup$




Cicad the singer is having a downtime in his career.



Over the millennia, Cicad sang anywhere from royal courts to farmers' cottages and have been rewarded with many, many hand-claps.
But to keep going, indeed to preserve his immortality, he must sing - in battle.



When leading a charge to meet the enemy, Cicad would raise his mighty voice and sword in song, inspire his troops - and his superpower. Yes, the secret behind his long life is twofold:



The first: Singing in battle makes him as strong as three men, and impervious to attack. Even a ballista bolt couldn't scratch him, so long as he's singing with at least two of his men within earshot, fighting against at least two of his enemies.



The second: Singing in battle at least once every couple of years keeps him from aging, thereby letting him live for millennia.



  • At least four men must be fighting, not including himself.

  • The intent to kill the other guys is what constitutes a fight, so they can use long range weapons, even cannons or missiles. However, the field must be such that his guys will have at least caught a glimpse of some of the enemy, at a point in time that's close to the time of the attack (firing a missile at people so far away that you can't ever see them doesn't answer the requirement of "a fight").

  • For the same reason, the fight can't be a mock fight (e.g. practice or a fun bar fight). An intent to kill is required.

  • Earshot means that at least two of his troops must be able to hear his song.

  • Cicad doesn't particularly like to fight, but he's good at it and the troops love him for both the songs and, well, the fact that his side tends to win. So every two years, at least one fight.

This was all well and good in the old days, when a good charge with melee weapons constituted a good fight.



Nowadays, however... There are plenty of fights to be had but not the kind where singing is appreciated. Indeed, as recently as the 19th century Cicad has faced frowns when singing while the battle rages.



In this day and age, where concentration and stealth is a prime concern in battle, he was threatened with removal from duty for singing at the wrong times. This almost made him come forth and tell of his abilities, but he absolutely does not want to be probed and bisected by scientists who'll want to check what makes his superpower tick.



So the question is:
What kind of situations in modern day combat warrant loud singing by at least one particular combatant? The bigger the advantage one side gets by the singing, the better the answer."



  • For the sake of the question, lets assume that other noises (e.g gun/cannon fire, screams of the dying) are not getting in the way of Cicad's singing.

Edit 1:
I really should have clarified this - I want Cicad to be running in the field and getting smacked with fire and brimstone (bullets and heavy ordinance) from time to time. So, infantry. He can be in a tank for a bit of the fight but the point of his imperiousness to harm is to get an unhinged singer into the thick of it. I'll worry about people not noticing his invincibility later...



Edit 2: For those still complaining about other noises, as I wrote - hand (ear?) wave it. Part of the superpower is that the singing subdues noises that are undesired by the audience (which can be a cool advantage by itself, just not so relevant to this question).







reality-check modern-age combat social-norms music






share|improve this question















share|improve this question













share|improve this question




share|improve this question








edited Aug 28 '18 at 7:46







Nahshon paz

















asked Aug 26 '18 at 13:11









Nahshon pazNahshon paz

3,23932040




3,23932040







  • 1




    $begingroup$
    Presumably he's an infantryman? (Because if he's Artillery, Armor, the Corps of Engineers or Service and Support Branches he won't be seeing much.)
    $endgroup$
    – RonJohn
    Aug 26 '18 at 17:47






  • 2




    $begingroup$
    It all depends on who's the leader of the club that's made for you and me.
    $endgroup$
    – Mazura
    Aug 26 '18 at 20:51










  • $begingroup$
    He should turn himself into an Amazee Dayzee. Those things‘ music is super annoying.
    $endgroup$
    – DonielF
    Aug 26 '18 at 23:59










  • $begingroup$
    The question could be broadened to ask in what situation in combat in any era it made sense for a warrior to sing. That uses oxygen and brain power which could better be employed in fighting. Musicians (particularly trumpeters and drummers) have at times had a rôle in boosting morale or communicating orders, but they didn't play with one hand and wield a weapon in the other.
    $endgroup$
    – Peter Taylor
    Aug 27 '18 at 7:47






  • 4




    $begingroup$
    This is a wonderful, wacky question and I love it. "I've been working on the..." BANG! "railroad! All the live long day!" WHACK! "I've been working..." GRUNT! GURGLE! RATATATAT! "working on the raaaailroadddd!" It's like the Joker joined the Marines.
    $endgroup$
    – JBH
    Aug 27 '18 at 19:22












  • 1




    $begingroup$
    Presumably he's an infantryman? (Because if he's Artillery, Armor, the Corps of Engineers or Service and Support Branches he won't be seeing much.)
    $endgroup$
    – RonJohn
    Aug 26 '18 at 17:47






  • 2




    $begingroup$
    It all depends on who's the leader of the club that's made for you and me.
    $endgroup$
    – Mazura
    Aug 26 '18 at 20:51










  • $begingroup$
    He should turn himself into an Amazee Dayzee. Those things‘ music is super annoying.
    $endgroup$
    – DonielF
    Aug 26 '18 at 23:59










  • $begingroup$
    The question could be broadened to ask in what situation in combat in any era it made sense for a warrior to sing. That uses oxygen and brain power which could better be employed in fighting. Musicians (particularly trumpeters and drummers) have at times had a rôle in boosting morale or communicating orders, but they didn't play with one hand and wield a weapon in the other.
    $endgroup$
    – Peter Taylor
    Aug 27 '18 at 7:47






  • 4




    $begingroup$
    This is a wonderful, wacky question and I love it. "I've been working on the..." BANG! "railroad! All the live long day!" WHACK! "I've been working..." GRUNT! GURGLE! RATATATAT! "working on the raaaailroadddd!" It's like the Joker joined the Marines.
    $endgroup$
    – JBH
    Aug 27 '18 at 19:22







1




1




$begingroup$
Presumably he's an infantryman? (Because if he's Artillery, Armor, the Corps of Engineers or Service and Support Branches he won't be seeing much.)
$endgroup$
– RonJohn
Aug 26 '18 at 17:47




$begingroup$
Presumably he's an infantryman? (Because if he's Artillery, Armor, the Corps of Engineers or Service and Support Branches he won't be seeing much.)
$endgroup$
– RonJohn
Aug 26 '18 at 17:47




2




2




$begingroup$
It all depends on who's the leader of the club that's made for you and me.
$endgroup$
– Mazura
Aug 26 '18 at 20:51




$begingroup$
It all depends on who's the leader of the club that's made for you and me.
$endgroup$
– Mazura
Aug 26 '18 at 20:51












$begingroup$
He should turn himself into an Amazee Dayzee. Those things‘ music is super annoying.
$endgroup$
– DonielF
Aug 26 '18 at 23:59




$begingroup$
He should turn himself into an Amazee Dayzee. Those things‘ music is super annoying.
$endgroup$
– DonielF
Aug 26 '18 at 23:59












$begingroup$
The question could be broadened to ask in what situation in combat in any era it made sense for a warrior to sing. That uses oxygen and brain power which could better be employed in fighting. Musicians (particularly trumpeters and drummers) have at times had a rôle in boosting morale or communicating orders, but they didn't play with one hand and wield a weapon in the other.
$endgroup$
– Peter Taylor
Aug 27 '18 at 7:47




$begingroup$
The question could be broadened to ask in what situation in combat in any era it made sense for a warrior to sing. That uses oxygen and brain power which could better be employed in fighting. Musicians (particularly trumpeters and drummers) have at times had a rôle in boosting morale or communicating orders, but they didn't play with one hand and wield a weapon in the other.
$endgroup$
– Peter Taylor
Aug 27 '18 at 7:47




4




4




$begingroup$
This is a wonderful, wacky question and I love it. "I've been working on the..." BANG! "railroad! All the live long day!" WHACK! "I've been working..." GRUNT! GURGLE! RATATATAT! "working on the raaaailroadddd!" It's like the Joker joined the Marines.
$endgroup$
– JBH
Aug 27 '18 at 19:22




$begingroup$
This is a wonderful, wacky question and I love it. "I've been working on the..." BANG! "railroad! All the live long day!" WHACK! "I've been working..." GRUNT! GURGLE! RATATATAT! "working on the raaaailroadddd!" It's like the Joker joined the Marines.
$endgroup$
– JBH
Aug 27 '18 at 19:22










10 Answers
10






active

oldest

votes


















17












$begingroup$

This answer is based on my own time in the military which included 2 tours:




What kind of situations in modern day combat warrant loud singing by at least one particular combatant?




There is no direct combat situation in a realistic battle which would allow for this.



The reason? Because your statement lets assume that other noises are not getting in the way is impossible in a normal situation. Meaning even if he could be heard, assuming a super power, it would still not be allowed by his squad. The reasons:



Combat is Loud



Imagine explosions, gun fire, screaming, and your own heart pounding in your ears - modern combat is incredibly loud. A quick overview of decibel values: 100 db is like a jackhammer, 120db is 4 times as loud and considered painful (e.g. a chain saw). 150db is 8 times as loud as that and is enough to rupture your ear drums (source). Now, imagine every vehicle running with 100 or more db, each weapon (from several directions!) firing with 145-190db and a Marines' own weapon blasting away 160db right in front of their face (see pages 79-81).



Indeed, it is so loud that the number 1 & 2 combat related injuries involve hearing loss. Military members can't even wear the current hearing protection we have because it makes it too hard to hear orders from you commanders and other fire team members.



Someone singing would just be more noise and never welcome during actual combat.



Many times your already wearing communication (comm) equipment



Currently there are a lot of different ideas on improving ones ability to hear in combat but most of these are still in the development stages.



Ever played a video game online and someone just keeps stepping on everyone trying to talk over a single comm channel? Now imagine your in combat and Cicad is singing over comm (so people can hear him) and you don't want to know what the squad will do to stop this person who is keeping them from hearing orders, calling med-evacs, and coordinating actions.



How could he sing in modern combat?



Well, he couldn't, not out loud with the typical situations you encounter in a real modern, urban, battlefield. However, the military is always researching new methods which could be modified for the sake of a story and there is one situation which is rare but happens.



  1. Singing used as a form of biofeedback

Currently the military is researching a whole lot of audio feedback methods for treating PTSD & also TBI, doing better at physical exercise and mental tasks, and to train military members for combat.



Cicad could participate in studies like these, which require him to record his voice for military members to hear in combat (if an effect is seen). This could actually eventually allow him to remove himself from combat if the recording of his voice works as well as him actually singing or he is sent to combat and required to sing due to improvements seen in these studies (which only happen with a "live performance").



  1. "Happy Birthday"

Yeah, believe or not even in the bloodiest battle in Iraq Marines stopped to sing their hymn on November 10th. If you only need once a year, this event could work as Cicad would only need to start singing the Marine Hymn and his squad would likely join in.



  1. "Paint a Target on my Back"

I have never actually seen this and don't believe most of the stories I've heard but: there is the myth of the guy who starts making noise on purpose, who just gets up and distracts the enemy so his squad can see where the enemy positions are. Suppressing fire is usually used but Cicad could start singing to distract the enemy or get them to expose their position so the rest of the squad could move in.






share|improve this answer











$endgroup$








  • 7




    $begingroup$
    That last point is especially salient, given that Cicad is impervious to attack while singing in battle. Who better to draw enemy fire?
    $endgroup$
    – Bobson
    Aug 27 '18 at 12:42






  • 1




    $begingroup$
    "each weapon is adding 145-190 decibels" - That part sounds like 10 weapons would create a sound of 1450-1900 decibel, which is impossible on earth. Otherwise, this is a realistic answer.
    $endgroup$
    – R. Schmitz
    Aug 27 '18 at 13:24






  • 4




    $begingroup$
    @RSchmitz It might sound that way, to people who don't know what a decibel is. It's a base-10 logarithmic scale. If your weapon is 150dB, then 10 weapons equals 160dB, 100 weapons equals 170dB, and so on.
    $endgroup$
    – Graham
    Aug 27 '18 at 23:49






  • 1




    $begingroup$
    “Paint a target on my back” Ever seen Firefly, episode The Message?
    $endgroup$
    – can-ned_food
    Aug 28 '18 at 5:41






  • 2




    $begingroup$
    Cheers @JGreenwell , I very much like the "Paint a Target on my Back" and "Happy Birthday" bits. Remember that Cicad is a bit loose in the head after several thousands of years, fighting (many "Happy" birthdays...). Perhaps he looses care for his life sometimes, and doesn't mind testing the limits of his powers. Or maybe he just likes taking the pain away from his soldiers. Your answer is the best one so mark, but as for "There is no direct combat situation in a realistic battle which would allow for this." - that's what WBuilding is all about :-)
    $endgroup$
    – Nahshon paz
    Aug 30 '18 at 7:06



















36












$begingroup$

Singing still has a place in modern battle.



https://www.ultimate-guitar.com/news/general_music_news/us_marines_blaring_acdc_to_agitate_iraqi_insurgents.html




According to the Associated Press, U.S. marines in Fallujah, Iraq have
been broadcasting messages by loudspeaker to agitate Iraqi insurgents,
announcing, "You are cowards for hiding behind women and children.
Come out and fight," and blaring heavy-metal music, including AC/DC's
"Shoot To Thrill".




I read an account of tank warfare in the first Iraq war where the guys in the tank were all singing along to "Thunder" (also AC/DC) as they went looking for opponents. Stealth is not a consideration when you are in a tank.



The place for singing in modern war is during city battles where stealth is not an issue, but especially during motorized or airborne expeditions where your vehicle is obvious. Singing is still good for morale.



It occurs to me that a drone pilot silo might be a fine place for singing also. Drone pilots are viewing the enemy (albeit remotely) so that meets OP criterion. The enemy can't hear you so no harm in singing. And it is freaking boring for long stretches of time and the songs will break up the monotony.






share|improve this answer









$endgroup$








  • 8




    $begingroup$
    Being in a tank or drone controller hut break the firing a missile at people so far away that you can't ever see them doesn't answer the requirement of "a fight" requirement*.
    $endgroup$
    – RonJohn
    Aug 26 '18 at 18:25










  • $begingroup$
    @RonJohn how do you think a tank crew picks their target?
    $endgroup$
    – Willk
    Aug 26 '18 at 20:10






  • 5




    $begingroup$
    Usually not directly with their eyes, and even if a tank commander sticks his head out to look around, he sees the enemy tank not the people inside the tank.
    $endgroup$
    – RonJohn
    Aug 26 '18 at 20:18






  • 5




    $begingroup$
    Also, I get the impression that Cicad the singer needs to be up close and personal.
    $endgroup$
    – RonJohn
    Aug 26 '18 at 20:19










  • $begingroup$
    Also, being in artillery would tend to be far enough away from the action, as well as loud enough on it's own, to allow singing. Being on a battleship and sending off rockets, while singing "Anchor's Away" might get you some backup singers. Besides, once you're in a firefight with dozens or hundreds of people firing around you, you're likely to have ear protection anyway, and then you can sing.
    $endgroup$
    – computercarguy
    Aug 27 '18 at 15:26


















20












$begingroup$

He's going about it all wrong if he's in a western military. Even most infantrymen rarely, if ever, see who they're shooting at. He should join a 3rd-world military as a mercenary. They have much looser discipline, meaning he won't be reprimanded for singing. Further, their equipment and training are much poorer, so they often need to get much closer to their targets to actually fight; meaning he WILL get to see his target.



Alternatively, he could be a vigilante. Fighting to the death is not strictly a military task. He could go hunt down narcoterrorists in Central or South America. Law enforcement wouldn't even try to stop him for the most part.






share|improve this answer











$endgroup$








  • 8




    $begingroup$
    I upvoted this, but am having second thoughts. "most infantrymen rarely, if ever, see who they're shooting at" sounds horribly wrong. How do you have fire discipline if you're just spraying and praying? How do you make sure you don't accidentally hit a non-combatant? "Know what you're shooting at" is Rule #1.
    $endgroup$
    – RonJohn
    Aug 26 '18 at 20:21






  • 11




    $begingroup$
    Most firing is in the general vicinity of a target, for suppressing fire. There's a reason US soldiers fire 250,000 rounds per enemy combatant killed in Afghanistan. belfasttelegraph.co.uk/news/world-news/… That reason is that in most gunfights, you just spray lead at cover the enemy is likely to be behind until you can call in something heavy enough to destroy that cover. It is quite likely that only one man in squad saw the enemy, and the rest just followed his lead.
    $endgroup$
    – Ryan_L
    Aug 26 '18 at 20:30






  • 3




    $begingroup$
    Building clearing is up-close and personal (true, having someone singing would be horrible in this situation so that could be an answer in itself). However, the idea that infantrymen rarely see their enemy (rarely face to face) is just wrong. Study the Battle of Fallujah and other modern battles - and you will see just how up close and personal modern urban warfare is. The idea of moving to specific law enforcement is a good point.
    $endgroup$
    – JGreenwell
    Aug 27 '18 at 1:27











  • $begingroup$
    Not all units end up doing building clearing. Further, even in a unit that does, if you are not in an entry team, again you will just be providing suppressing fire at likely enemy positions.
    $endgroup$
    – Ryan_L
    Aug 27 '18 at 3:23






  • 1




    $begingroup$
    entry team, extraction team, MPs who got called up because you just don't have enough grunts, dang it we were just transporting goods and got caught in the cross-fire in the streets supply team, etc. Building clearing is just one part of the experience of IOF/OEF - IEDs were another and those either went off and you saw nobody or you were taking direct fire quick (and yeah, really close). If suppressing fire you are already close enough to see the enemy and calling in a strike can depend on a whole lot of conditions (like is it on an oil field, is there a sandstorm, are they already running)
    $endgroup$
    – JGreenwell
    Aug 27 '18 at 11:11


















7












$begingroup$


I give my orders through limerick

it may sound weird, but there's a trick

with the rhymes so clear

all the men will hear

through the radio static thin and thick!




I got the idea from reading @Wolfgang's answer. As a radio operator myself I know how hard it can be to hear things through static and I think rhymes would really help with that.



I went with a 9-9-5-5-9 form limerick. Feel free to edit my answer to add more limericks that fit the topic. :)






share|improve this answer









$endgroup$












  • $begingroup$
    Sounds like Battle Language from Herbert's Dune books.
    $endgroup$
    – can-ned_food
    Aug 28 '18 at 5:43






  • 2




    $begingroup$
    This was used by the mongols so orders would not be repeated incorrectly.
    $endgroup$
    – PStag
    Aug 28 '18 at 10:12






  • 1




    $begingroup$
    @PStag I don't know about the Mongols specifically, but rhyme and other linguistic tricks have been used by storytellers in cultures with oral traditions to ensure that they remember and pass down legends correctly.
    $endgroup$
    – Kapten-N
    Aug 28 '18 at 12:17










  • $begingroup$
    I guess that's a partial solution. He won't always be able to issue commands as songs.
    $endgroup$
    – Nahshon paz
    Aug 29 '18 at 6:07


















6












$begingroup$

What about non-military? He could join a local gang if he can find one to fit whatever his moral code is. Or start his own gang. I don't have any personal experience, but it seems that at that level, you're (at least sometimes) back down to the traditional knife, club, and fist.






share|improve this answer









$endgroup$












  • $begingroup$
    That can be part of the story but for the most part, he's an honorable (if somewhat unhinged) infantry charger.
    $endgroup$
    – Nahshon paz
    Aug 29 '18 at 5:57


















6












$begingroup$

https://en.wikipedia.org/wiki/Jack_Churchill



This guy used a sword and a bow and arrow along with a pair of bagpipes in world war 2. I think the idea was to demoralise the enemy. It's probably a case of if you can get the job done while singing then go right ahead and sing. He might get a bad reputation at first but after he has pulled of a few heroic actions people will start to respect him despite his oddity.






share|improve this answer









$endgroup$












  • $begingroup$
    Glad somebody finally mentioned Mad Jack!
    $endgroup$
    – chrylis
    Aug 28 '18 at 10:37






  • 1




    $begingroup$
    +1, but several answers explain how modern warfare is quite different from Word War II.
    $endgroup$
    – Pere
    Aug 28 '18 at 15:19


















3












$begingroup$

Timing. Cicad is a somewhat eccentric infantry officer that uses song to coordinate assaults. His men think its strange, but it gets the mission accomplished. This could be used over short range radio as well. Assaults in an urban setting would exemplify this well, with different parts of the song coordinating different actions of the platoon members.






share|improve this answer









$endgroup$




















    3












    $begingroup$

    Same old, same old...to boost morale, increase hormone levels and pulse to sharpen senses and raise euphoria / aggression, tighten team bonding and focus (stronger together)



    I'd love to list some points of the following but it is just too much tl;dr info.



    • An interesting study about singing along to (in this case western pop) music:
      http://www.doc.gold.ac.uk/~mas03dm/papers/PawleyMullensiefen_Singalong_2012.pdf


    • music / sound as a weapon:
      https://www.newyorker.com/magazine/2016/07/04/when-music-is-violence
      The New Yorker - July 4, 2016, issue, with the headline “The Sound of Hate.”


    • see also the role singing played in the World War I Christmas Truce of 1914


    • Stanford University psychologists Scott S. Wiltermuth and Chip Heath conducted a series of experiments to see how synchronous movement affects group interactions.[...]These findings suggest that cultural practices which involve synchrony (such as dancing, singing or marching) may enable groups to produce members who are cooperative and willing to make personal sacrifices, for the benefit of the group.(Journal Reference:
      Synchrony and Cooperation. Psychological Science, January 2009)


    • there are quite a few videos on youtube showing russian soldiers marching while chanting the theme song to spongebob squarepants (;
      https://www.youtube.com/watch?v=vhuzb3WMntc





    share|improve this answer











    $endgroup$




















      2












      $begingroup$


      Right that moment I was feeling unusually expendable, almost expended, because I was hearing the sweetest sound in the universe, the beacon the retrieval boat would land on, sounding our recall. The beacon is a fast insertion ship, fired ahead of the retrieval boat, containing a single crew member that starts singing that welcome, welcome music. The retrieval boat homes in on it automatically three minutes later and you had better be on hand, because the bus can’t wait and there won’t be another one along.



      But you don’t walk away on another cap trooper, not while there’s a chance he’s still alive — not in Rasczak’s Roughnecks. Not in any outfit of the Mobile Infantry. You try to make pickup.



      I heard Jelly order: "Fleads up, lads! Close to retrieval circle and interdict! On the bounce!"



      And I heard the Battle-singer Cicad’s sweet voice: "— to the everlasting glory of the infantry, shines the name, shines the name of Rodger Young!" and I wanted to head for it so bad I could taste it.




      Robert A. Heinlein, Starship Troopers, modified.



      In this modern day and age, Cicad plays a central role in in his platoon, due to his unique skills. Every mission ends with his Retrieval Team serving as a beacon for the landing troops to gather and prepare for extraction, his legendary presence and powerful voice serving as a boost to the Infantry troops.






      share|improve this answer









      $endgroup$




















        0












        $begingroup$

        The "Dynamo" character in Schwarzenegger's "Running Man" was singing before he started zapping people. That's a bit in the future, but opportunities for a murderous singing assassin appearing on TV might arise.






        share|improve this answer









        $endgroup$








        • 1




          $begingroup$
          Seems to me rather far from the premise of the question. That was an arena being shown for entertainment, and the Runners were meant to be unarmed.
          $endgroup$
          – can-ned_food
          Aug 28 '18 at 5:45










        Your Answer





        StackExchange.ifUsing("editor", function ()
        return StackExchange.using("mathjaxEditing", function ()
        StackExchange.MarkdownEditor.creationCallbacks.add(function (editor, postfix)
        StackExchange.mathjaxEditing.prepareWmdForMathJax(editor, postfix, [["$", "$"], ["\\(","\\)"]]);
        );
        );
        , "mathjax-editing");

        StackExchange.ready(function()
        var channelOptions =
        tags: "".split(" "),
        id: "579"
        ;
        initTagRenderer("".split(" "), "".split(" "), channelOptions);

        StackExchange.using("externalEditor", function()
        // Have to fire editor after snippets, if snippets enabled
        if (StackExchange.settings.snippets.snippetsEnabled)
        StackExchange.using("snippets", function()
        createEditor();
        );

        else
        createEditor();

        );

        function createEditor()
        StackExchange.prepareEditor(
        heartbeatType: 'answer',
        autoActivateHeartbeat: false,
        convertImagesToLinks: false,
        noModals: true,
        showLowRepImageUploadWarning: true,
        reputationToPostImages: null,
        bindNavPrevention: true,
        postfix: "",
        imageUploader:
        brandingHtml: "Powered by u003ca class="icon-imgur-white" href="https://imgur.com/"u003eu003c/au003e",
        contentPolicyHtml: "User contributions licensed under u003ca href="https://creativecommons.org/licenses/by-sa/3.0/"u003ecc by-sa 3.0 with attribution requiredu003c/au003e u003ca href="https://stackoverflow.com/legal/content-policy"u003e(content policy)u003c/au003e",
        allowUrls: true
        ,
        noCode: true, onDemand: true,
        discardSelector: ".discard-answer"
        ,immediatelyShowMarkdownHelp:true
        );



        );













        draft saved

        draft discarded


















        StackExchange.ready(
        function ()
        StackExchange.openid.initPostLogin('.new-post-login', 'https%3a%2f%2fworldbuilding.stackexchange.com%2fquestions%2f123060%2fin-modern-day-combat-what-would-warrant-singing-during-battle%23new-answer', 'question_page');

        );

        Post as a guest















        Required, but never shown

























        10 Answers
        10






        active

        oldest

        votes








        10 Answers
        10






        active

        oldest

        votes









        active

        oldest

        votes






        active

        oldest

        votes









        17












        $begingroup$

        This answer is based on my own time in the military which included 2 tours:




        What kind of situations in modern day combat warrant loud singing by at least one particular combatant?




        There is no direct combat situation in a realistic battle which would allow for this.



        The reason? Because your statement lets assume that other noises are not getting in the way is impossible in a normal situation. Meaning even if he could be heard, assuming a super power, it would still not be allowed by his squad. The reasons:



        Combat is Loud



        Imagine explosions, gun fire, screaming, and your own heart pounding in your ears - modern combat is incredibly loud. A quick overview of decibel values: 100 db is like a jackhammer, 120db is 4 times as loud and considered painful (e.g. a chain saw). 150db is 8 times as loud as that and is enough to rupture your ear drums (source). Now, imagine every vehicle running with 100 or more db, each weapon (from several directions!) firing with 145-190db and a Marines' own weapon blasting away 160db right in front of their face (see pages 79-81).



        Indeed, it is so loud that the number 1 & 2 combat related injuries involve hearing loss. Military members can't even wear the current hearing protection we have because it makes it too hard to hear orders from you commanders and other fire team members.



        Someone singing would just be more noise and never welcome during actual combat.



        Many times your already wearing communication (comm) equipment



        Currently there are a lot of different ideas on improving ones ability to hear in combat but most of these are still in the development stages.



        Ever played a video game online and someone just keeps stepping on everyone trying to talk over a single comm channel? Now imagine your in combat and Cicad is singing over comm (so people can hear him) and you don't want to know what the squad will do to stop this person who is keeping them from hearing orders, calling med-evacs, and coordinating actions.



        How could he sing in modern combat?



        Well, he couldn't, not out loud with the typical situations you encounter in a real modern, urban, battlefield. However, the military is always researching new methods which could be modified for the sake of a story and there is one situation which is rare but happens.



        1. Singing used as a form of biofeedback

        Currently the military is researching a whole lot of audio feedback methods for treating PTSD & also TBI, doing better at physical exercise and mental tasks, and to train military members for combat.



        Cicad could participate in studies like these, which require him to record his voice for military members to hear in combat (if an effect is seen). This could actually eventually allow him to remove himself from combat if the recording of his voice works as well as him actually singing or he is sent to combat and required to sing due to improvements seen in these studies (which only happen with a "live performance").



        1. "Happy Birthday"

        Yeah, believe or not even in the bloodiest battle in Iraq Marines stopped to sing their hymn on November 10th. If you only need once a year, this event could work as Cicad would only need to start singing the Marine Hymn and his squad would likely join in.



        1. "Paint a Target on my Back"

        I have never actually seen this and don't believe most of the stories I've heard but: there is the myth of the guy who starts making noise on purpose, who just gets up and distracts the enemy so his squad can see where the enemy positions are. Suppressing fire is usually used but Cicad could start singing to distract the enemy or get them to expose their position so the rest of the squad could move in.






        share|improve this answer











        $endgroup$








        • 7




          $begingroup$
          That last point is especially salient, given that Cicad is impervious to attack while singing in battle. Who better to draw enemy fire?
          $endgroup$
          – Bobson
          Aug 27 '18 at 12:42






        • 1




          $begingroup$
          "each weapon is adding 145-190 decibels" - That part sounds like 10 weapons would create a sound of 1450-1900 decibel, which is impossible on earth. Otherwise, this is a realistic answer.
          $endgroup$
          – R. Schmitz
          Aug 27 '18 at 13:24






        • 4




          $begingroup$
          @RSchmitz It might sound that way, to people who don't know what a decibel is. It's a base-10 logarithmic scale. If your weapon is 150dB, then 10 weapons equals 160dB, 100 weapons equals 170dB, and so on.
          $endgroup$
          – Graham
          Aug 27 '18 at 23:49






        • 1




          $begingroup$
          “Paint a target on my back” Ever seen Firefly, episode The Message?
          $endgroup$
          – can-ned_food
          Aug 28 '18 at 5:41






        • 2




          $begingroup$
          Cheers @JGreenwell , I very much like the "Paint a Target on my Back" and "Happy Birthday" bits. Remember that Cicad is a bit loose in the head after several thousands of years, fighting (many "Happy" birthdays...). Perhaps he looses care for his life sometimes, and doesn't mind testing the limits of his powers. Or maybe he just likes taking the pain away from his soldiers. Your answer is the best one so mark, but as for "There is no direct combat situation in a realistic battle which would allow for this." - that's what WBuilding is all about :-)
          $endgroup$
          – Nahshon paz
          Aug 30 '18 at 7:06
















        17












        $begingroup$

        This answer is based on my own time in the military which included 2 tours:




        What kind of situations in modern day combat warrant loud singing by at least one particular combatant?




        There is no direct combat situation in a realistic battle which would allow for this.



        The reason? Because your statement lets assume that other noises are not getting in the way is impossible in a normal situation. Meaning even if he could be heard, assuming a super power, it would still not be allowed by his squad. The reasons:



        Combat is Loud



        Imagine explosions, gun fire, screaming, and your own heart pounding in your ears - modern combat is incredibly loud. A quick overview of decibel values: 100 db is like a jackhammer, 120db is 4 times as loud and considered painful (e.g. a chain saw). 150db is 8 times as loud as that and is enough to rupture your ear drums (source). Now, imagine every vehicle running with 100 or more db, each weapon (from several directions!) firing with 145-190db and a Marines' own weapon blasting away 160db right in front of their face (see pages 79-81).



        Indeed, it is so loud that the number 1 & 2 combat related injuries involve hearing loss. Military members can't even wear the current hearing protection we have because it makes it too hard to hear orders from you commanders and other fire team members.



        Someone singing would just be more noise and never welcome during actual combat.



        Many times your already wearing communication (comm) equipment



        Currently there are a lot of different ideas on improving ones ability to hear in combat but most of these are still in the development stages.



        Ever played a video game online and someone just keeps stepping on everyone trying to talk over a single comm channel? Now imagine your in combat and Cicad is singing over comm (so people can hear him) and you don't want to know what the squad will do to stop this person who is keeping them from hearing orders, calling med-evacs, and coordinating actions.



        How could he sing in modern combat?



        Well, he couldn't, not out loud with the typical situations you encounter in a real modern, urban, battlefield. However, the military is always researching new methods which could be modified for the sake of a story and there is one situation which is rare but happens.



        1. Singing used as a form of biofeedback

        Currently the military is researching a whole lot of audio feedback methods for treating PTSD & also TBI, doing better at physical exercise and mental tasks, and to train military members for combat.



        Cicad could participate in studies like these, which require him to record his voice for military members to hear in combat (if an effect is seen). This could actually eventually allow him to remove himself from combat if the recording of his voice works as well as him actually singing or he is sent to combat and required to sing due to improvements seen in these studies (which only happen with a "live performance").



        1. "Happy Birthday"

        Yeah, believe or not even in the bloodiest battle in Iraq Marines stopped to sing their hymn on November 10th. If you only need once a year, this event could work as Cicad would only need to start singing the Marine Hymn and his squad would likely join in.



        1. "Paint a Target on my Back"

        I have never actually seen this and don't believe most of the stories I've heard but: there is the myth of the guy who starts making noise on purpose, who just gets up and distracts the enemy so his squad can see where the enemy positions are. Suppressing fire is usually used but Cicad could start singing to distract the enemy or get them to expose their position so the rest of the squad could move in.






        share|improve this answer











        $endgroup$








        • 7




          $begingroup$
          That last point is especially salient, given that Cicad is impervious to attack while singing in battle. Who better to draw enemy fire?
          $endgroup$
          – Bobson
          Aug 27 '18 at 12:42






        • 1




          $begingroup$
          "each weapon is adding 145-190 decibels" - That part sounds like 10 weapons would create a sound of 1450-1900 decibel, which is impossible on earth. Otherwise, this is a realistic answer.
          $endgroup$
          – R. Schmitz
          Aug 27 '18 at 13:24






        • 4




          $begingroup$
          @RSchmitz It might sound that way, to people who don't know what a decibel is. It's a base-10 logarithmic scale. If your weapon is 150dB, then 10 weapons equals 160dB, 100 weapons equals 170dB, and so on.
          $endgroup$
          – Graham
          Aug 27 '18 at 23:49






        • 1




          $begingroup$
          “Paint a target on my back” Ever seen Firefly, episode The Message?
          $endgroup$
          – can-ned_food
          Aug 28 '18 at 5:41






        • 2




          $begingroup$
          Cheers @JGreenwell , I very much like the "Paint a Target on my Back" and "Happy Birthday" bits. Remember that Cicad is a bit loose in the head after several thousands of years, fighting (many "Happy" birthdays...). Perhaps he looses care for his life sometimes, and doesn't mind testing the limits of his powers. Or maybe he just likes taking the pain away from his soldiers. Your answer is the best one so mark, but as for "There is no direct combat situation in a realistic battle which would allow for this." - that's what WBuilding is all about :-)
          $endgroup$
          – Nahshon paz
          Aug 30 '18 at 7:06














        17












        17








        17





        $begingroup$

        This answer is based on my own time in the military which included 2 tours:




        What kind of situations in modern day combat warrant loud singing by at least one particular combatant?




        There is no direct combat situation in a realistic battle which would allow for this.



        The reason? Because your statement lets assume that other noises are not getting in the way is impossible in a normal situation. Meaning even if he could be heard, assuming a super power, it would still not be allowed by his squad. The reasons:



        Combat is Loud



        Imagine explosions, gun fire, screaming, and your own heart pounding in your ears - modern combat is incredibly loud. A quick overview of decibel values: 100 db is like a jackhammer, 120db is 4 times as loud and considered painful (e.g. a chain saw). 150db is 8 times as loud as that and is enough to rupture your ear drums (source). Now, imagine every vehicle running with 100 or more db, each weapon (from several directions!) firing with 145-190db and a Marines' own weapon blasting away 160db right in front of their face (see pages 79-81).



        Indeed, it is so loud that the number 1 & 2 combat related injuries involve hearing loss. Military members can't even wear the current hearing protection we have because it makes it too hard to hear orders from you commanders and other fire team members.



        Someone singing would just be more noise and never welcome during actual combat.



        Many times your already wearing communication (comm) equipment



        Currently there are a lot of different ideas on improving ones ability to hear in combat but most of these are still in the development stages.



        Ever played a video game online and someone just keeps stepping on everyone trying to talk over a single comm channel? Now imagine your in combat and Cicad is singing over comm (so people can hear him) and you don't want to know what the squad will do to stop this person who is keeping them from hearing orders, calling med-evacs, and coordinating actions.



        How could he sing in modern combat?



        Well, he couldn't, not out loud with the typical situations you encounter in a real modern, urban, battlefield. However, the military is always researching new methods which could be modified for the sake of a story and there is one situation which is rare but happens.



        1. Singing used as a form of biofeedback

        Currently the military is researching a whole lot of audio feedback methods for treating PTSD & also TBI, doing better at physical exercise and mental tasks, and to train military members for combat.



        Cicad could participate in studies like these, which require him to record his voice for military members to hear in combat (if an effect is seen). This could actually eventually allow him to remove himself from combat if the recording of his voice works as well as him actually singing or he is sent to combat and required to sing due to improvements seen in these studies (which only happen with a "live performance").



        1. "Happy Birthday"

        Yeah, believe or not even in the bloodiest battle in Iraq Marines stopped to sing their hymn on November 10th. If you only need once a year, this event could work as Cicad would only need to start singing the Marine Hymn and his squad would likely join in.



        1. "Paint a Target on my Back"

        I have never actually seen this and don't believe most of the stories I've heard but: there is the myth of the guy who starts making noise on purpose, who just gets up and distracts the enemy so his squad can see where the enemy positions are. Suppressing fire is usually used but Cicad could start singing to distract the enemy or get them to expose their position so the rest of the squad could move in.






        share|improve this answer











        $endgroup$



        This answer is based on my own time in the military which included 2 tours:




        What kind of situations in modern day combat warrant loud singing by at least one particular combatant?




        There is no direct combat situation in a realistic battle which would allow for this.



        The reason? Because your statement lets assume that other noises are not getting in the way is impossible in a normal situation. Meaning even if he could be heard, assuming a super power, it would still not be allowed by his squad. The reasons:



        Combat is Loud



        Imagine explosions, gun fire, screaming, and your own heart pounding in your ears - modern combat is incredibly loud. A quick overview of decibel values: 100 db is like a jackhammer, 120db is 4 times as loud and considered painful (e.g. a chain saw). 150db is 8 times as loud as that and is enough to rupture your ear drums (source). Now, imagine every vehicle running with 100 or more db, each weapon (from several directions!) firing with 145-190db and a Marines' own weapon blasting away 160db right in front of their face (see pages 79-81).



        Indeed, it is so loud that the number 1 & 2 combat related injuries involve hearing loss. Military members can't even wear the current hearing protection we have because it makes it too hard to hear orders from you commanders and other fire team members.



        Someone singing would just be more noise and never welcome during actual combat.



        Many times your already wearing communication (comm) equipment



        Currently there are a lot of different ideas on improving ones ability to hear in combat but most of these are still in the development stages.



        Ever played a video game online and someone just keeps stepping on everyone trying to talk over a single comm channel? Now imagine your in combat and Cicad is singing over comm (so people can hear him) and you don't want to know what the squad will do to stop this person who is keeping them from hearing orders, calling med-evacs, and coordinating actions.



        How could he sing in modern combat?



        Well, he couldn't, not out loud with the typical situations you encounter in a real modern, urban, battlefield. However, the military is always researching new methods which could be modified for the sake of a story and there is one situation which is rare but happens.



        1. Singing used as a form of biofeedback

        Currently the military is researching a whole lot of audio feedback methods for treating PTSD & also TBI, doing better at physical exercise and mental tasks, and to train military members for combat.



        Cicad could participate in studies like these, which require him to record his voice for military members to hear in combat (if an effect is seen). This could actually eventually allow him to remove himself from combat if the recording of his voice works as well as him actually singing or he is sent to combat and required to sing due to improvements seen in these studies (which only happen with a "live performance").



        1. "Happy Birthday"

        Yeah, believe or not even in the bloodiest battle in Iraq Marines stopped to sing their hymn on November 10th. If you only need once a year, this event could work as Cicad would only need to start singing the Marine Hymn and his squad would likely join in.



        1. "Paint a Target on my Back"

        I have never actually seen this and don't believe most of the stories I've heard but: there is the myth of the guy who starts making noise on purpose, who just gets up and distracts the enemy so his squad can see where the enemy positions are. Suppressing fire is usually used but Cicad could start singing to distract the enemy or get them to expose their position so the rest of the squad could move in.







        share|improve this answer














        share|improve this answer



        share|improve this answer








        edited Aug 27 '18 at 14:58









        R. Schmitz

        25316




        25316










        answered Aug 27 '18 at 3:14









        JGreenwellJGreenwell

        1,320215




        1,320215







        • 7




          $begingroup$
          That last point is especially salient, given that Cicad is impervious to attack while singing in battle. Who better to draw enemy fire?
          $endgroup$
          – Bobson
          Aug 27 '18 at 12:42






        • 1




          $begingroup$
          "each weapon is adding 145-190 decibels" - That part sounds like 10 weapons would create a sound of 1450-1900 decibel, which is impossible on earth. Otherwise, this is a realistic answer.
          $endgroup$
          – R. Schmitz
          Aug 27 '18 at 13:24






        • 4




          $begingroup$
          @RSchmitz It might sound that way, to people who don't know what a decibel is. It's a base-10 logarithmic scale. If your weapon is 150dB, then 10 weapons equals 160dB, 100 weapons equals 170dB, and so on.
          $endgroup$
          – Graham
          Aug 27 '18 at 23:49






        • 1




          $begingroup$
          “Paint a target on my back” Ever seen Firefly, episode The Message?
          $endgroup$
          – can-ned_food
          Aug 28 '18 at 5:41






        • 2




          $begingroup$
          Cheers @JGreenwell , I very much like the "Paint a Target on my Back" and "Happy Birthday" bits. Remember that Cicad is a bit loose in the head after several thousands of years, fighting (many "Happy" birthdays...). Perhaps he looses care for his life sometimes, and doesn't mind testing the limits of his powers. Or maybe he just likes taking the pain away from his soldiers. Your answer is the best one so mark, but as for "There is no direct combat situation in a realistic battle which would allow for this." - that's what WBuilding is all about :-)
          $endgroup$
          – Nahshon paz
          Aug 30 '18 at 7:06













        • 7




          $begingroup$
          That last point is especially salient, given that Cicad is impervious to attack while singing in battle. Who better to draw enemy fire?
          $endgroup$
          – Bobson
          Aug 27 '18 at 12:42






        • 1




          $begingroup$
          "each weapon is adding 145-190 decibels" - That part sounds like 10 weapons would create a sound of 1450-1900 decibel, which is impossible on earth. Otherwise, this is a realistic answer.
          $endgroup$
          – R. Schmitz
          Aug 27 '18 at 13:24






        • 4




          $begingroup$
          @RSchmitz It might sound that way, to people who don't know what a decibel is. It's a base-10 logarithmic scale. If your weapon is 150dB, then 10 weapons equals 160dB, 100 weapons equals 170dB, and so on.
          $endgroup$
          – Graham
          Aug 27 '18 at 23:49






        • 1




          $begingroup$
          “Paint a target on my back” Ever seen Firefly, episode The Message?
          $endgroup$
          – can-ned_food
          Aug 28 '18 at 5:41






        • 2




          $begingroup$
          Cheers @JGreenwell , I very much like the "Paint a Target on my Back" and "Happy Birthday" bits. Remember that Cicad is a bit loose in the head after several thousands of years, fighting (many "Happy" birthdays...). Perhaps he looses care for his life sometimes, and doesn't mind testing the limits of his powers. Or maybe he just likes taking the pain away from his soldiers. Your answer is the best one so mark, but as for "There is no direct combat situation in a realistic battle which would allow for this." - that's what WBuilding is all about :-)
          $endgroup$
          – Nahshon paz
          Aug 30 '18 at 7:06








        7




        7




        $begingroup$
        That last point is especially salient, given that Cicad is impervious to attack while singing in battle. Who better to draw enemy fire?
        $endgroup$
        – Bobson
        Aug 27 '18 at 12:42




        $begingroup$
        That last point is especially salient, given that Cicad is impervious to attack while singing in battle. Who better to draw enemy fire?
        $endgroup$
        – Bobson
        Aug 27 '18 at 12:42




        1




        1




        $begingroup$
        "each weapon is adding 145-190 decibels" - That part sounds like 10 weapons would create a sound of 1450-1900 decibel, which is impossible on earth. Otherwise, this is a realistic answer.
        $endgroup$
        – R. Schmitz
        Aug 27 '18 at 13:24




        $begingroup$
        "each weapon is adding 145-190 decibels" - That part sounds like 10 weapons would create a sound of 1450-1900 decibel, which is impossible on earth. Otherwise, this is a realistic answer.
        $endgroup$
        – R. Schmitz
        Aug 27 '18 at 13:24




        4




        4




        $begingroup$
        @RSchmitz It might sound that way, to people who don't know what a decibel is. It's a base-10 logarithmic scale. If your weapon is 150dB, then 10 weapons equals 160dB, 100 weapons equals 170dB, and so on.
        $endgroup$
        – Graham
        Aug 27 '18 at 23:49




        $begingroup$
        @RSchmitz It might sound that way, to people who don't know what a decibel is. It's a base-10 logarithmic scale. If your weapon is 150dB, then 10 weapons equals 160dB, 100 weapons equals 170dB, and so on.
        $endgroup$
        – Graham
        Aug 27 '18 at 23:49




        1




        1




        $begingroup$
        “Paint a target on my back” Ever seen Firefly, episode The Message?
        $endgroup$
        – can-ned_food
        Aug 28 '18 at 5:41




        $begingroup$
        “Paint a target on my back” Ever seen Firefly, episode The Message?
        $endgroup$
        – can-ned_food
        Aug 28 '18 at 5:41




        2




        2




        $begingroup$
        Cheers @JGreenwell , I very much like the "Paint a Target on my Back" and "Happy Birthday" bits. Remember that Cicad is a bit loose in the head after several thousands of years, fighting (many "Happy" birthdays...). Perhaps he looses care for his life sometimes, and doesn't mind testing the limits of his powers. Or maybe he just likes taking the pain away from his soldiers. Your answer is the best one so mark, but as for "There is no direct combat situation in a realistic battle which would allow for this." - that's what WBuilding is all about :-)
        $endgroup$
        – Nahshon paz
        Aug 30 '18 at 7:06





        $begingroup$
        Cheers @JGreenwell , I very much like the "Paint a Target on my Back" and "Happy Birthday" bits. Remember that Cicad is a bit loose in the head after several thousands of years, fighting (many "Happy" birthdays...). Perhaps he looses care for his life sometimes, and doesn't mind testing the limits of his powers. Or maybe he just likes taking the pain away from his soldiers. Your answer is the best one so mark, but as for "There is no direct combat situation in a realistic battle which would allow for this." - that's what WBuilding is all about :-)
        $endgroup$
        – Nahshon paz
        Aug 30 '18 at 7:06












        36












        $begingroup$

        Singing still has a place in modern battle.



        https://www.ultimate-guitar.com/news/general_music_news/us_marines_blaring_acdc_to_agitate_iraqi_insurgents.html




        According to the Associated Press, U.S. marines in Fallujah, Iraq have
        been broadcasting messages by loudspeaker to agitate Iraqi insurgents,
        announcing, "You are cowards for hiding behind women and children.
        Come out and fight," and blaring heavy-metal music, including AC/DC's
        "Shoot To Thrill".




        I read an account of tank warfare in the first Iraq war where the guys in the tank were all singing along to "Thunder" (also AC/DC) as they went looking for opponents. Stealth is not a consideration when you are in a tank.



        The place for singing in modern war is during city battles where stealth is not an issue, but especially during motorized or airborne expeditions where your vehicle is obvious. Singing is still good for morale.



        It occurs to me that a drone pilot silo might be a fine place for singing also. Drone pilots are viewing the enemy (albeit remotely) so that meets OP criterion. The enemy can't hear you so no harm in singing. And it is freaking boring for long stretches of time and the songs will break up the monotony.






        share|improve this answer









        $endgroup$








        • 8




          $begingroup$
          Being in a tank or drone controller hut break the firing a missile at people so far away that you can't ever see them doesn't answer the requirement of "a fight" requirement*.
          $endgroup$
          – RonJohn
          Aug 26 '18 at 18:25










        • $begingroup$
          @RonJohn how do you think a tank crew picks their target?
          $endgroup$
          – Willk
          Aug 26 '18 at 20:10






        • 5




          $begingroup$
          Usually not directly with their eyes, and even if a tank commander sticks his head out to look around, he sees the enemy tank not the people inside the tank.
          $endgroup$
          – RonJohn
          Aug 26 '18 at 20:18






        • 5




          $begingroup$
          Also, I get the impression that Cicad the singer needs to be up close and personal.
          $endgroup$
          – RonJohn
          Aug 26 '18 at 20:19










        • $begingroup$
          Also, being in artillery would tend to be far enough away from the action, as well as loud enough on it's own, to allow singing. Being on a battleship and sending off rockets, while singing "Anchor's Away" might get you some backup singers. Besides, once you're in a firefight with dozens or hundreds of people firing around you, you're likely to have ear protection anyway, and then you can sing.
          $endgroup$
          – computercarguy
          Aug 27 '18 at 15:26















        36












        $begingroup$

        Singing still has a place in modern battle.



        https://www.ultimate-guitar.com/news/general_music_news/us_marines_blaring_acdc_to_agitate_iraqi_insurgents.html




        According to the Associated Press, U.S. marines in Fallujah, Iraq have
        been broadcasting messages by loudspeaker to agitate Iraqi insurgents,
        announcing, "You are cowards for hiding behind women and children.
        Come out and fight," and blaring heavy-metal music, including AC/DC's
        "Shoot To Thrill".




        I read an account of tank warfare in the first Iraq war where the guys in the tank were all singing along to "Thunder" (also AC/DC) as they went looking for opponents. Stealth is not a consideration when you are in a tank.



        The place for singing in modern war is during city battles where stealth is not an issue, but especially during motorized or airborne expeditions where your vehicle is obvious. Singing is still good for morale.



        It occurs to me that a drone pilot silo might be a fine place for singing also. Drone pilots are viewing the enemy (albeit remotely) so that meets OP criterion. The enemy can't hear you so no harm in singing. And it is freaking boring for long stretches of time and the songs will break up the monotony.






        share|improve this answer









        $endgroup$








        • 8




          $begingroup$
          Being in a tank or drone controller hut break the firing a missile at people so far away that you can't ever see them doesn't answer the requirement of "a fight" requirement*.
          $endgroup$
          – RonJohn
          Aug 26 '18 at 18:25










        • $begingroup$
          @RonJohn how do you think a tank crew picks their target?
          $endgroup$
          – Willk
          Aug 26 '18 at 20:10






        • 5




          $begingroup$
          Usually not directly with their eyes, and even if a tank commander sticks his head out to look around, he sees the enemy tank not the people inside the tank.
          $endgroup$
          – RonJohn
          Aug 26 '18 at 20:18






        • 5




          $begingroup$
          Also, I get the impression that Cicad the singer needs to be up close and personal.
          $endgroup$
          – RonJohn
          Aug 26 '18 at 20:19










        • $begingroup$
          Also, being in artillery would tend to be far enough away from the action, as well as loud enough on it's own, to allow singing. Being on a battleship and sending off rockets, while singing "Anchor's Away" might get you some backup singers. Besides, once you're in a firefight with dozens or hundreds of people firing around you, you're likely to have ear protection anyway, and then you can sing.
          $endgroup$
          – computercarguy
          Aug 27 '18 at 15:26













        36












        36








        36





        $begingroup$

        Singing still has a place in modern battle.



        https://www.ultimate-guitar.com/news/general_music_news/us_marines_blaring_acdc_to_agitate_iraqi_insurgents.html




        According to the Associated Press, U.S. marines in Fallujah, Iraq have
        been broadcasting messages by loudspeaker to agitate Iraqi insurgents,
        announcing, "You are cowards for hiding behind women and children.
        Come out and fight," and blaring heavy-metal music, including AC/DC's
        "Shoot To Thrill".




        I read an account of tank warfare in the first Iraq war where the guys in the tank were all singing along to "Thunder" (also AC/DC) as they went looking for opponents. Stealth is not a consideration when you are in a tank.



        The place for singing in modern war is during city battles where stealth is not an issue, but especially during motorized or airborne expeditions where your vehicle is obvious. Singing is still good for morale.



        It occurs to me that a drone pilot silo might be a fine place for singing also. Drone pilots are viewing the enemy (albeit remotely) so that meets OP criterion. The enemy can't hear you so no harm in singing. And it is freaking boring for long stretches of time and the songs will break up the monotony.






        share|improve this answer









        $endgroup$



        Singing still has a place in modern battle.



        https://www.ultimate-guitar.com/news/general_music_news/us_marines_blaring_acdc_to_agitate_iraqi_insurgents.html




        According to the Associated Press, U.S. marines in Fallujah, Iraq have
        been broadcasting messages by loudspeaker to agitate Iraqi insurgents,
        announcing, "You are cowards for hiding behind women and children.
        Come out and fight," and blaring heavy-metal music, including AC/DC's
        "Shoot To Thrill".




        I read an account of tank warfare in the first Iraq war where the guys in the tank were all singing along to "Thunder" (also AC/DC) as they went looking for opponents. Stealth is not a consideration when you are in a tank.



        The place for singing in modern war is during city battles where stealth is not an issue, but especially during motorized or airborne expeditions where your vehicle is obvious. Singing is still good for morale.



        It occurs to me that a drone pilot silo might be a fine place for singing also. Drone pilots are viewing the enemy (albeit remotely) so that meets OP criterion. The enemy can't hear you so no harm in singing. And it is freaking boring for long stretches of time and the songs will break up the monotony.







        share|improve this answer












        share|improve this answer



        share|improve this answer










        answered Aug 26 '18 at 15:32









        WillkWillk

        106k26199446




        106k26199446







        • 8




          $begingroup$
          Being in a tank or drone controller hut break the firing a missile at people so far away that you can't ever see them doesn't answer the requirement of "a fight" requirement*.
          $endgroup$
          – RonJohn
          Aug 26 '18 at 18:25










        • $begingroup$
          @RonJohn how do you think a tank crew picks their target?
          $endgroup$
          – Willk
          Aug 26 '18 at 20:10






        • 5




          $begingroup$
          Usually not directly with their eyes, and even if a tank commander sticks his head out to look around, he sees the enemy tank not the people inside the tank.
          $endgroup$
          – RonJohn
          Aug 26 '18 at 20:18






        • 5




          $begingroup$
          Also, I get the impression that Cicad the singer needs to be up close and personal.
          $endgroup$
          – RonJohn
          Aug 26 '18 at 20:19










        • $begingroup$
          Also, being in artillery would tend to be far enough away from the action, as well as loud enough on it's own, to allow singing. Being on a battleship and sending off rockets, while singing "Anchor's Away" might get you some backup singers. Besides, once you're in a firefight with dozens or hundreds of people firing around you, you're likely to have ear protection anyway, and then you can sing.
          $endgroup$
          – computercarguy
          Aug 27 '18 at 15:26












        • 8




          $begingroup$
          Being in a tank or drone controller hut break the firing a missile at people so far away that you can't ever see them doesn't answer the requirement of "a fight" requirement*.
          $endgroup$
          – RonJohn
          Aug 26 '18 at 18:25










        • $begingroup$
          @RonJohn how do you think a tank crew picks their target?
          $endgroup$
          – Willk
          Aug 26 '18 at 20:10






        • 5




          $begingroup$
          Usually not directly with their eyes, and even if a tank commander sticks his head out to look around, he sees the enemy tank not the people inside the tank.
          $endgroup$
          – RonJohn
          Aug 26 '18 at 20:18






        • 5




          $begingroup$
          Also, I get the impression that Cicad the singer needs to be up close and personal.
          $endgroup$
          – RonJohn
          Aug 26 '18 at 20:19










        • $begingroup$
          Also, being in artillery would tend to be far enough away from the action, as well as loud enough on it's own, to allow singing. Being on a battleship and sending off rockets, while singing "Anchor's Away" might get you some backup singers. Besides, once you're in a firefight with dozens or hundreds of people firing around you, you're likely to have ear protection anyway, and then you can sing.
          $endgroup$
          – computercarguy
          Aug 27 '18 at 15:26







        8




        8




        $begingroup$
        Being in a tank or drone controller hut break the firing a missile at people so far away that you can't ever see them doesn't answer the requirement of "a fight" requirement*.
        $endgroup$
        – RonJohn
        Aug 26 '18 at 18:25




        $begingroup$
        Being in a tank or drone controller hut break the firing a missile at people so far away that you can't ever see them doesn't answer the requirement of "a fight" requirement*.
        $endgroup$
        – RonJohn
        Aug 26 '18 at 18:25












        $begingroup$
        @RonJohn how do you think a tank crew picks their target?
        $endgroup$
        – Willk
        Aug 26 '18 at 20:10




        $begingroup$
        @RonJohn how do you think a tank crew picks their target?
        $endgroup$
        – Willk
        Aug 26 '18 at 20:10




        5




        5




        $begingroup$
        Usually not directly with their eyes, and even if a tank commander sticks his head out to look around, he sees the enemy tank not the people inside the tank.
        $endgroup$
        – RonJohn
        Aug 26 '18 at 20:18




        $begingroup$
        Usually not directly with their eyes, and even if a tank commander sticks his head out to look around, he sees the enemy tank not the people inside the tank.
        $endgroup$
        – RonJohn
        Aug 26 '18 at 20:18




        5




        5




        $begingroup$
        Also, I get the impression that Cicad the singer needs to be up close and personal.
        $endgroup$
        – RonJohn
        Aug 26 '18 at 20:19




        $begingroup$
        Also, I get the impression that Cicad the singer needs to be up close and personal.
        $endgroup$
        – RonJohn
        Aug 26 '18 at 20:19












        $begingroup$
        Also, being in artillery would tend to be far enough away from the action, as well as loud enough on it's own, to allow singing. Being on a battleship and sending off rockets, while singing "Anchor's Away" might get you some backup singers. Besides, once you're in a firefight with dozens or hundreds of people firing around you, you're likely to have ear protection anyway, and then you can sing.
        $endgroup$
        – computercarguy
        Aug 27 '18 at 15:26




        $begingroup$
        Also, being in artillery would tend to be far enough away from the action, as well as loud enough on it's own, to allow singing. Being on a battleship and sending off rockets, while singing "Anchor's Away" might get you some backup singers. Besides, once you're in a firefight with dozens or hundreds of people firing around you, you're likely to have ear protection anyway, and then you can sing.
        $endgroup$
        – computercarguy
        Aug 27 '18 at 15:26











        20












        $begingroup$

        He's going about it all wrong if he's in a western military. Even most infantrymen rarely, if ever, see who they're shooting at. He should join a 3rd-world military as a mercenary. They have much looser discipline, meaning he won't be reprimanded for singing. Further, their equipment and training are much poorer, so they often need to get much closer to their targets to actually fight; meaning he WILL get to see his target.



        Alternatively, he could be a vigilante. Fighting to the death is not strictly a military task. He could go hunt down narcoterrorists in Central or South America. Law enforcement wouldn't even try to stop him for the most part.






        share|improve this answer











        $endgroup$








        • 8




          $begingroup$
          I upvoted this, but am having second thoughts. "most infantrymen rarely, if ever, see who they're shooting at" sounds horribly wrong. How do you have fire discipline if you're just spraying and praying? How do you make sure you don't accidentally hit a non-combatant? "Know what you're shooting at" is Rule #1.
          $endgroup$
          – RonJohn
          Aug 26 '18 at 20:21






        • 11




          $begingroup$
          Most firing is in the general vicinity of a target, for suppressing fire. There's a reason US soldiers fire 250,000 rounds per enemy combatant killed in Afghanistan. belfasttelegraph.co.uk/news/world-news/… That reason is that in most gunfights, you just spray lead at cover the enemy is likely to be behind until you can call in something heavy enough to destroy that cover. It is quite likely that only one man in squad saw the enemy, and the rest just followed his lead.
          $endgroup$
          – Ryan_L
          Aug 26 '18 at 20:30






        • 3




          $begingroup$
          Building clearing is up-close and personal (true, having someone singing would be horrible in this situation so that could be an answer in itself). However, the idea that infantrymen rarely see their enemy (rarely face to face) is just wrong. Study the Battle of Fallujah and other modern battles - and you will see just how up close and personal modern urban warfare is. The idea of moving to specific law enforcement is a good point.
          $endgroup$
          – JGreenwell
          Aug 27 '18 at 1:27











        • $begingroup$
          Not all units end up doing building clearing. Further, even in a unit that does, if you are not in an entry team, again you will just be providing suppressing fire at likely enemy positions.
          $endgroup$
          – Ryan_L
          Aug 27 '18 at 3:23






        • 1




          $begingroup$
          entry team, extraction team, MPs who got called up because you just don't have enough grunts, dang it we were just transporting goods and got caught in the cross-fire in the streets supply team, etc. Building clearing is just one part of the experience of IOF/OEF - IEDs were another and those either went off and you saw nobody or you were taking direct fire quick (and yeah, really close). If suppressing fire you are already close enough to see the enemy and calling in a strike can depend on a whole lot of conditions (like is it on an oil field, is there a sandstorm, are they already running)
          $endgroup$
          – JGreenwell
          Aug 27 '18 at 11:11















        20












        $begingroup$

        He's going about it all wrong if he's in a western military. Even most infantrymen rarely, if ever, see who they're shooting at. He should join a 3rd-world military as a mercenary. They have much looser discipline, meaning he won't be reprimanded for singing. Further, their equipment and training are much poorer, so they often need to get much closer to their targets to actually fight; meaning he WILL get to see his target.



        Alternatively, he could be a vigilante. Fighting to the death is not strictly a military task. He could go hunt down narcoterrorists in Central or South America. Law enforcement wouldn't even try to stop him for the most part.






        share|improve this answer











        $endgroup$








        • 8




          $begingroup$
          I upvoted this, but am having second thoughts. "most infantrymen rarely, if ever, see who they're shooting at" sounds horribly wrong. How do you have fire discipline if you're just spraying and praying? How do you make sure you don't accidentally hit a non-combatant? "Know what you're shooting at" is Rule #1.
          $endgroup$
          – RonJohn
          Aug 26 '18 at 20:21






        • 11




          $begingroup$
          Most firing is in the general vicinity of a target, for suppressing fire. There's a reason US soldiers fire 250,000 rounds per enemy combatant killed in Afghanistan. belfasttelegraph.co.uk/news/world-news/… That reason is that in most gunfights, you just spray lead at cover the enemy is likely to be behind until you can call in something heavy enough to destroy that cover. It is quite likely that only one man in squad saw the enemy, and the rest just followed his lead.
          $endgroup$
          – Ryan_L
          Aug 26 '18 at 20:30






        • 3




          $begingroup$
          Building clearing is up-close and personal (true, having someone singing would be horrible in this situation so that could be an answer in itself). However, the idea that infantrymen rarely see their enemy (rarely face to face) is just wrong. Study the Battle of Fallujah and other modern battles - and you will see just how up close and personal modern urban warfare is. The idea of moving to specific law enforcement is a good point.
          $endgroup$
          – JGreenwell
          Aug 27 '18 at 1:27











        • $begingroup$
          Not all units end up doing building clearing. Further, even in a unit that does, if you are not in an entry team, again you will just be providing suppressing fire at likely enemy positions.
          $endgroup$
          – Ryan_L
          Aug 27 '18 at 3:23






        • 1




          $begingroup$
          entry team, extraction team, MPs who got called up because you just don't have enough grunts, dang it we were just transporting goods and got caught in the cross-fire in the streets supply team, etc. Building clearing is just one part of the experience of IOF/OEF - IEDs were another and those either went off and you saw nobody or you were taking direct fire quick (and yeah, really close). If suppressing fire you are already close enough to see the enemy and calling in a strike can depend on a whole lot of conditions (like is it on an oil field, is there a sandstorm, are they already running)
          $endgroup$
          – JGreenwell
          Aug 27 '18 at 11:11













        20












        20








        20





        $begingroup$

        He's going about it all wrong if he's in a western military. Even most infantrymen rarely, if ever, see who they're shooting at. He should join a 3rd-world military as a mercenary. They have much looser discipline, meaning he won't be reprimanded for singing. Further, their equipment and training are much poorer, so they often need to get much closer to their targets to actually fight; meaning he WILL get to see his target.



        Alternatively, he could be a vigilante. Fighting to the death is not strictly a military task. He could go hunt down narcoterrorists in Central or South America. Law enforcement wouldn't even try to stop him for the most part.






        share|improve this answer











        $endgroup$



        He's going about it all wrong if he's in a western military. Even most infantrymen rarely, if ever, see who they're shooting at. He should join a 3rd-world military as a mercenary. They have much looser discipline, meaning he won't be reprimanded for singing. Further, their equipment and training are much poorer, so they often need to get much closer to their targets to actually fight; meaning he WILL get to see his target.



        Alternatively, he could be a vigilante. Fighting to the death is not strictly a military task. He could go hunt down narcoterrorists in Central or South America. Law enforcement wouldn't even try to stop him for the most part.







        share|improve this answer














        share|improve this answer



        share|improve this answer








        edited Aug 28 '18 at 5:26









        can-ned_food

        1,5371824




        1,5371824










        answered Aug 26 '18 at 18:18









        Ryan_LRyan_L

        4,340925




        4,340925







        • 8




          $begingroup$
          I upvoted this, but am having second thoughts. "most infantrymen rarely, if ever, see who they're shooting at" sounds horribly wrong. How do you have fire discipline if you're just spraying and praying? How do you make sure you don't accidentally hit a non-combatant? "Know what you're shooting at" is Rule #1.
          $endgroup$
          – RonJohn
          Aug 26 '18 at 20:21






        • 11




          $begingroup$
          Most firing is in the general vicinity of a target, for suppressing fire. There's a reason US soldiers fire 250,000 rounds per enemy combatant killed in Afghanistan. belfasttelegraph.co.uk/news/world-news/… That reason is that in most gunfights, you just spray lead at cover the enemy is likely to be behind until you can call in something heavy enough to destroy that cover. It is quite likely that only one man in squad saw the enemy, and the rest just followed his lead.
          $endgroup$
          – Ryan_L
          Aug 26 '18 at 20:30






        • 3




          $begingroup$
          Building clearing is up-close and personal (true, having someone singing would be horrible in this situation so that could be an answer in itself). However, the idea that infantrymen rarely see their enemy (rarely face to face) is just wrong. Study the Battle of Fallujah and other modern battles - and you will see just how up close and personal modern urban warfare is. The idea of moving to specific law enforcement is a good point.
          $endgroup$
          – JGreenwell
          Aug 27 '18 at 1:27











        • $begingroup$
          Not all units end up doing building clearing. Further, even in a unit that does, if you are not in an entry team, again you will just be providing suppressing fire at likely enemy positions.
          $endgroup$
          – Ryan_L
          Aug 27 '18 at 3:23






        • 1




          $begingroup$
          entry team, extraction team, MPs who got called up because you just don't have enough grunts, dang it we were just transporting goods and got caught in the cross-fire in the streets supply team, etc. Building clearing is just one part of the experience of IOF/OEF - IEDs were another and those either went off and you saw nobody or you were taking direct fire quick (and yeah, really close). If suppressing fire you are already close enough to see the enemy and calling in a strike can depend on a whole lot of conditions (like is it on an oil field, is there a sandstorm, are they already running)
          $endgroup$
          – JGreenwell
          Aug 27 '18 at 11:11












        • 8




          $begingroup$
          I upvoted this, but am having second thoughts. "most infantrymen rarely, if ever, see who they're shooting at" sounds horribly wrong. How do you have fire discipline if you're just spraying and praying? How do you make sure you don't accidentally hit a non-combatant? "Know what you're shooting at" is Rule #1.
          $endgroup$
          – RonJohn
          Aug 26 '18 at 20:21






        • 11




          $begingroup$
          Most firing is in the general vicinity of a target, for suppressing fire. There's a reason US soldiers fire 250,000 rounds per enemy combatant killed in Afghanistan. belfasttelegraph.co.uk/news/world-news/… That reason is that in most gunfights, you just spray lead at cover the enemy is likely to be behind until you can call in something heavy enough to destroy that cover. It is quite likely that only one man in squad saw the enemy, and the rest just followed his lead.
          $endgroup$
          – Ryan_L
          Aug 26 '18 at 20:30






        • 3




          $begingroup$
          Building clearing is up-close and personal (true, having someone singing would be horrible in this situation so that could be an answer in itself). However, the idea that infantrymen rarely see their enemy (rarely face to face) is just wrong. Study the Battle of Fallujah and other modern battles - and you will see just how up close and personal modern urban warfare is. The idea of moving to specific law enforcement is a good point.
          $endgroup$
          – JGreenwell
          Aug 27 '18 at 1:27











        • $begingroup$
          Not all units end up doing building clearing. Further, even in a unit that does, if you are not in an entry team, again you will just be providing suppressing fire at likely enemy positions.
          $endgroup$
          – Ryan_L
          Aug 27 '18 at 3:23






        • 1




          $begingroup$
          entry team, extraction team, MPs who got called up because you just don't have enough grunts, dang it we were just transporting goods and got caught in the cross-fire in the streets supply team, etc. Building clearing is just one part of the experience of IOF/OEF - IEDs were another and those either went off and you saw nobody or you were taking direct fire quick (and yeah, really close). If suppressing fire you are already close enough to see the enemy and calling in a strike can depend on a whole lot of conditions (like is it on an oil field, is there a sandstorm, are they already running)
          $endgroup$
          – JGreenwell
          Aug 27 '18 at 11:11







        8




        8




        $begingroup$
        I upvoted this, but am having second thoughts. "most infantrymen rarely, if ever, see who they're shooting at" sounds horribly wrong. How do you have fire discipline if you're just spraying and praying? How do you make sure you don't accidentally hit a non-combatant? "Know what you're shooting at" is Rule #1.
        $endgroup$
        – RonJohn
        Aug 26 '18 at 20:21




        $begingroup$
        I upvoted this, but am having second thoughts. "most infantrymen rarely, if ever, see who they're shooting at" sounds horribly wrong. How do you have fire discipline if you're just spraying and praying? How do you make sure you don't accidentally hit a non-combatant? "Know what you're shooting at" is Rule #1.
        $endgroup$
        – RonJohn
        Aug 26 '18 at 20:21




        11




        11




        $begingroup$
        Most firing is in the general vicinity of a target, for suppressing fire. There's a reason US soldiers fire 250,000 rounds per enemy combatant killed in Afghanistan. belfasttelegraph.co.uk/news/world-news/… That reason is that in most gunfights, you just spray lead at cover the enemy is likely to be behind until you can call in something heavy enough to destroy that cover. It is quite likely that only one man in squad saw the enemy, and the rest just followed his lead.
        $endgroup$
        – Ryan_L
        Aug 26 '18 at 20:30




        $begingroup$
        Most firing is in the general vicinity of a target, for suppressing fire. There's a reason US soldiers fire 250,000 rounds per enemy combatant killed in Afghanistan. belfasttelegraph.co.uk/news/world-news/… That reason is that in most gunfights, you just spray lead at cover the enemy is likely to be behind until you can call in something heavy enough to destroy that cover. It is quite likely that only one man in squad saw the enemy, and the rest just followed his lead.
        $endgroup$
        – Ryan_L
        Aug 26 '18 at 20:30




        3




        3




        $begingroup$
        Building clearing is up-close and personal (true, having someone singing would be horrible in this situation so that could be an answer in itself). However, the idea that infantrymen rarely see their enemy (rarely face to face) is just wrong. Study the Battle of Fallujah and other modern battles - and you will see just how up close and personal modern urban warfare is. The idea of moving to specific law enforcement is a good point.
        $endgroup$
        – JGreenwell
        Aug 27 '18 at 1:27





        $begingroup$
        Building clearing is up-close and personal (true, having someone singing would be horrible in this situation so that could be an answer in itself). However, the idea that infantrymen rarely see their enemy (rarely face to face) is just wrong. Study the Battle of Fallujah and other modern battles - and you will see just how up close and personal modern urban warfare is. The idea of moving to specific law enforcement is a good point.
        $endgroup$
        – JGreenwell
        Aug 27 '18 at 1:27













        $begingroup$
        Not all units end up doing building clearing. Further, even in a unit that does, if you are not in an entry team, again you will just be providing suppressing fire at likely enemy positions.
        $endgroup$
        – Ryan_L
        Aug 27 '18 at 3:23




        $begingroup$
        Not all units end up doing building clearing. Further, even in a unit that does, if you are not in an entry team, again you will just be providing suppressing fire at likely enemy positions.
        $endgroup$
        – Ryan_L
        Aug 27 '18 at 3:23




        1




        1




        $begingroup$
        entry team, extraction team, MPs who got called up because you just don't have enough grunts, dang it we were just transporting goods and got caught in the cross-fire in the streets supply team, etc. Building clearing is just one part of the experience of IOF/OEF - IEDs were another and those either went off and you saw nobody or you were taking direct fire quick (and yeah, really close). If suppressing fire you are already close enough to see the enemy and calling in a strike can depend on a whole lot of conditions (like is it on an oil field, is there a sandstorm, are they already running)
        $endgroup$
        – JGreenwell
        Aug 27 '18 at 11:11




        $begingroup$
        entry team, extraction team, MPs who got called up because you just don't have enough grunts, dang it we were just transporting goods and got caught in the cross-fire in the streets supply team, etc. Building clearing is just one part of the experience of IOF/OEF - IEDs were another and those either went off and you saw nobody or you were taking direct fire quick (and yeah, really close). If suppressing fire you are already close enough to see the enemy and calling in a strike can depend on a whole lot of conditions (like is it on an oil field, is there a sandstorm, are they already running)
        $endgroup$
        – JGreenwell
        Aug 27 '18 at 11:11











        7












        $begingroup$


        I give my orders through limerick

        it may sound weird, but there's a trick

        with the rhymes so clear

        all the men will hear

        through the radio static thin and thick!




        I got the idea from reading @Wolfgang's answer. As a radio operator myself I know how hard it can be to hear things through static and I think rhymes would really help with that.



        I went with a 9-9-5-5-9 form limerick. Feel free to edit my answer to add more limericks that fit the topic. :)






        share|improve this answer









        $endgroup$












        • $begingroup$
          Sounds like Battle Language from Herbert's Dune books.
          $endgroup$
          – can-ned_food
          Aug 28 '18 at 5:43






        • 2




          $begingroup$
          This was used by the mongols so orders would not be repeated incorrectly.
          $endgroup$
          – PStag
          Aug 28 '18 at 10:12






        • 1




          $begingroup$
          @PStag I don't know about the Mongols specifically, but rhyme and other linguistic tricks have been used by storytellers in cultures with oral traditions to ensure that they remember and pass down legends correctly.
          $endgroup$
          – Kapten-N
          Aug 28 '18 at 12:17










        • $begingroup$
          I guess that's a partial solution. He won't always be able to issue commands as songs.
          $endgroup$
          – Nahshon paz
          Aug 29 '18 at 6:07















        7












        $begingroup$


        I give my orders through limerick

        it may sound weird, but there's a trick

        with the rhymes so clear

        all the men will hear

        through the radio static thin and thick!




        I got the idea from reading @Wolfgang's answer. As a radio operator myself I know how hard it can be to hear things through static and I think rhymes would really help with that.



        I went with a 9-9-5-5-9 form limerick. Feel free to edit my answer to add more limericks that fit the topic. :)






        share|improve this answer









        $endgroup$












        • $begingroup$
          Sounds like Battle Language from Herbert's Dune books.
          $endgroup$
          – can-ned_food
          Aug 28 '18 at 5:43






        • 2




          $begingroup$
          This was used by the mongols so orders would not be repeated incorrectly.
          $endgroup$
          – PStag
          Aug 28 '18 at 10:12






        • 1




          $begingroup$
          @PStag I don't know about the Mongols specifically, but rhyme and other linguistic tricks have been used by storytellers in cultures with oral traditions to ensure that they remember and pass down legends correctly.
          $endgroup$
          – Kapten-N
          Aug 28 '18 at 12:17










        • $begingroup$
          I guess that's a partial solution. He won't always be able to issue commands as songs.
          $endgroup$
          – Nahshon paz
          Aug 29 '18 at 6:07













        7












        7








        7





        $begingroup$


        I give my orders through limerick

        it may sound weird, but there's a trick

        with the rhymes so clear

        all the men will hear

        through the radio static thin and thick!




        I got the idea from reading @Wolfgang's answer. As a radio operator myself I know how hard it can be to hear things through static and I think rhymes would really help with that.



        I went with a 9-9-5-5-9 form limerick. Feel free to edit my answer to add more limericks that fit the topic. :)






        share|improve this answer









        $endgroup$




        I give my orders through limerick

        it may sound weird, but there's a trick

        with the rhymes so clear

        all the men will hear

        through the radio static thin and thick!




        I got the idea from reading @Wolfgang's answer. As a radio operator myself I know how hard it can be to hear things through static and I think rhymes would really help with that.



        I went with a 9-9-5-5-9 form limerick. Feel free to edit my answer to add more limericks that fit the topic. :)







        share|improve this answer












        share|improve this answer



        share|improve this answer










        answered Aug 27 '18 at 9:24









        Kapten-NKapten-N

        19314




        19314











        • $begingroup$
          Sounds like Battle Language from Herbert's Dune books.
          $endgroup$
          – can-ned_food
          Aug 28 '18 at 5:43






        • 2




          $begingroup$
          This was used by the mongols so orders would not be repeated incorrectly.
          $endgroup$
          – PStag
          Aug 28 '18 at 10:12






        • 1




          $begingroup$
          @PStag I don't know about the Mongols specifically, but rhyme and other linguistic tricks have been used by storytellers in cultures with oral traditions to ensure that they remember and pass down legends correctly.
          $endgroup$
          – Kapten-N
          Aug 28 '18 at 12:17










        • $begingroup$
          I guess that's a partial solution. He won't always be able to issue commands as songs.
          $endgroup$
          – Nahshon paz
          Aug 29 '18 at 6:07
















        • $begingroup$
          Sounds like Battle Language from Herbert's Dune books.
          $endgroup$
          – can-ned_food
          Aug 28 '18 at 5:43






        • 2




          $begingroup$
          This was used by the mongols so orders would not be repeated incorrectly.
          $endgroup$
          – PStag
          Aug 28 '18 at 10:12






        • 1




          $begingroup$
          @PStag I don't know about the Mongols specifically, but rhyme and other linguistic tricks have been used by storytellers in cultures with oral traditions to ensure that they remember and pass down legends correctly.
          $endgroup$
          – Kapten-N
          Aug 28 '18 at 12:17










        • $begingroup$
          I guess that's a partial solution. He won't always be able to issue commands as songs.
          $endgroup$
          – Nahshon paz
          Aug 29 '18 at 6:07















        $begingroup$
        Sounds like Battle Language from Herbert's Dune books.
        $endgroup$
        – can-ned_food
        Aug 28 '18 at 5:43




        $begingroup$
        Sounds like Battle Language from Herbert's Dune books.
        $endgroup$
        – can-ned_food
        Aug 28 '18 at 5:43




        2




        2




        $begingroup$
        This was used by the mongols so orders would not be repeated incorrectly.
        $endgroup$
        – PStag
        Aug 28 '18 at 10:12




        $begingroup$
        This was used by the mongols so orders would not be repeated incorrectly.
        $endgroup$
        – PStag
        Aug 28 '18 at 10:12




        1




        1




        $begingroup$
        @PStag I don't know about the Mongols specifically, but rhyme and other linguistic tricks have been used by storytellers in cultures with oral traditions to ensure that they remember and pass down legends correctly.
        $endgroup$
        – Kapten-N
        Aug 28 '18 at 12:17




        $begingroup$
        @PStag I don't know about the Mongols specifically, but rhyme and other linguistic tricks have been used by storytellers in cultures with oral traditions to ensure that they remember and pass down legends correctly.
        $endgroup$
        – Kapten-N
        Aug 28 '18 at 12:17












        $begingroup$
        I guess that's a partial solution. He won't always be able to issue commands as songs.
        $endgroup$
        – Nahshon paz
        Aug 29 '18 at 6:07




        $begingroup$
        I guess that's a partial solution. He won't always be able to issue commands as songs.
        $endgroup$
        – Nahshon paz
        Aug 29 '18 at 6:07











        6












        $begingroup$

        What about non-military? He could join a local gang if he can find one to fit whatever his moral code is. Or start his own gang. I don't have any personal experience, but it seems that at that level, you're (at least sometimes) back down to the traditional knife, club, and fist.






        share|improve this answer









        $endgroup$












        • $begingroup$
          That can be part of the story but for the most part, he's an honorable (if somewhat unhinged) infantry charger.
          $endgroup$
          – Nahshon paz
          Aug 29 '18 at 5:57















        6












        $begingroup$

        What about non-military? He could join a local gang if he can find one to fit whatever his moral code is. Or start his own gang. I don't have any personal experience, but it seems that at that level, you're (at least sometimes) back down to the traditional knife, club, and fist.






        share|improve this answer









        $endgroup$












        • $begingroup$
          That can be part of the story but for the most part, he's an honorable (if somewhat unhinged) infantry charger.
          $endgroup$
          – Nahshon paz
          Aug 29 '18 at 5:57













        6












        6








        6





        $begingroup$

        What about non-military? He could join a local gang if he can find one to fit whatever his moral code is. Or start his own gang. I don't have any personal experience, but it seems that at that level, you're (at least sometimes) back down to the traditional knife, club, and fist.






        share|improve this answer









        $endgroup$



        What about non-military? He could join a local gang if he can find one to fit whatever his moral code is. Or start his own gang. I don't have any personal experience, but it seems that at that level, you're (at least sometimes) back down to the traditional knife, club, and fist.







        share|improve this answer












        share|improve this answer



        share|improve this answer










        answered Aug 27 '18 at 2:16









        user34314user34314

        811




        811











        • $begingroup$
          That can be part of the story but for the most part, he's an honorable (if somewhat unhinged) infantry charger.
          $endgroup$
          – Nahshon paz
          Aug 29 '18 at 5:57
















        • $begingroup$
          That can be part of the story but for the most part, he's an honorable (if somewhat unhinged) infantry charger.
          $endgroup$
          – Nahshon paz
          Aug 29 '18 at 5:57















        $begingroup$
        That can be part of the story but for the most part, he's an honorable (if somewhat unhinged) infantry charger.
        $endgroup$
        – Nahshon paz
        Aug 29 '18 at 5:57




        $begingroup$
        That can be part of the story but for the most part, he's an honorable (if somewhat unhinged) infantry charger.
        $endgroup$
        – Nahshon paz
        Aug 29 '18 at 5:57











        6












        $begingroup$

        https://en.wikipedia.org/wiki/Jack_Churchill



        This guy used a sword and a bow and arrow along with a pair of bagpipes in world war 2. I think the idea was to demoralise the enemy. It's probably a case of if you can get the job done while singing then go right ahead and sing. He might get a bad reputation at first but after he has pulled of a few heroic actions people will start to respect him despite his oddity.






        share|improve this answer









        $endgroup$












        • $begingroup$
          Glad somebody finally mentioned Mad Jack!
          $endgroup$
          – chrylis
          Aug 28 '18 at 10:37






        • 1




          $begingroup$
          +1, but several answers explain how modern warfare is quite different from Word War II.
          $endgroup$
          – Pere
          Aug 28 '18 at 15:19















        6












        $begingroup$

        https://en.wikipedia.org/wiki/Jack_Churchill



        This guy used a sword and a bow and arrow along with a pair of bagpipes in world war 2. I think the idea was to demoralise the enemy. It's probably a case of if you can get the job done while singing then go right ahead and sing. He might get a bad reputation at first but after he has pulled of a few heroic actions people will start to respect him despite his oddity.






        share|improve this answer









        $endgroup$












        • $begingroup$
          Glad somebody finally mentioned Mad Jack!
          $endgroup$
          – chrylis
          Aug 28 '18 at 10:37






        • 1




          $begingroup$
          +1, but several answers explain how modern warfare is quite different from Word War II.
          $endgroup$
          – Pere
          Aug 28 '18 at 15:19













        6












        6








        6





        $begingroup$

        https://en.wikipedia.org/wiki/Jack_Churchill



        This guy used a sword and a bow and arrow along with a pair of bagpipes in world war 2. I think the idea was to demoralise the enemy. It's probably a case of if you can get the job done while singing then go right ahead and sing. He might get a bad reputation at first but after he has pulled of a few heroic actions people will start to respect him despite his oddity.






        share|improve this answer









        $endgroup$



        https://en.wikipedia.org/wiki/Jack_Churchill



        This guy used a sword and a bow and arrow along with a pair of bagpipes in world war 2. I think the idea was to demoralise the enemy. It's probably a case of if you can get the job done while singing then go right ahead and sing. He might get a bad reputation at first but after he has pulled of a few heroic actions people will start to respect him despite his oddity.







        share|improve this answer












        share|improve this answer



        share|improve this answer










        answered Aug 28 '18 at 10:20









        PStagPStag

        1,728816




        1,728816











        • $begingroup$
          Glad somebody finally mentioned Mad Jack!
          $endgroup$
          – chrylis
          Aug 28 '18 at 10:37






        • 1




          $begingroup$
          +1, but several answers explain how modern warfare is quite different from Word War II.
          $endgroup$
          – Pere
          Aug 28 '18 at 15:19
















        • $begingroup$
          Glad somebody finally mentioned Mad Jack!
          $endgroup$
          – chrylis
          Aug 28 '18 at 10:37






        • 1




          $begingroup$
          +1, but several answers explain how modern warfare is quite different from Word War II.
          $endgroup$
          – Pere
          Aug 28 '18 at 15:19















        $begingroup$
        Glad somebody finally mentioned Mad Jack!
        $endgroup$
        – chrylis
        Aug 28 '18 at 10:37




        $begingroup$
        Glad somebody finally mentioned Mad Jack!
        $endgroup$
        – chrylis
        Aug 28 '18 at 10:37




        1




        1




        $begingroup$
        +1, but several answers explain how modern warfare is quite different from Word War II.
        $endgroup$
        – Pere
        Aug 28 '18 at 15:19




        $begingroup$
        +1, but several answers explain how modern warfare is quite different from Word War II.
        $endgroup$
        – Pere
        Aug 28 '18 at 15:19











        3












        $begingroup$

        Timing. Cicad is a somewhat eccentric infantry officer that uses song to coordinate assaults. His men think its strange, but it gets the mission accomplished. This could be used over short range radio as well. Assaults in an urban setting would exemplify this well, with different parts of the song coordinating different actions of the platoon members.






        share|improve this answer









        $endgroup$

















          3












          $begingroup$

          Timing. Cicad is a somewhat eccentric infantry officer that uses song to coordinate assaults. His men think its strange, but it gets the mission accomplished. This could be used over short range radio as well. Assaults in an urban setting would exemplify this well, with different parts of the song coordinating different actions of the platoon members.






          share|improve this answer









          $endgroup$















            3












            3








            3





            $begingroup$

            Timing. Cicad is a somewhat eccentric infantry officer that uses song to coordinate assaults. His men think its strange, but it gets the mission accomplished. This could be used over short range radio as well. Assaults in an urban setting would exemplify this well, with different parts of the song coordinating different actions of the platoon members.






            share|improve this answer









            $endgroup$



            Timing. Cicad is a somewhat eccentric infantry officer that uses song to coordinate assaults. His men think its strange, but it gets the mission accomplished. This could be used over short range radio as well. Assaults in an urban setting would exemplify this well, with different parts of the song coordinating different actions of the platoon members.







            share|improve this answer












            share|improve this answer



            share|improve this answer










            answered Aug 27 '18 at 2:22









            WolfgangWolfgang

            49126




            49126





















                3












                $begingroup$

                Same old, same old...to boost morale, increase hormone levels and pulse to sharpen senses and raise euphoria / aggression, tighten team bonding and focus (stronger together)



                I'd love to list some points of the following but it is just too much tl;dr info.



                • An interesting study about singing along to (in this case western pop) music:
                  http://www.doc.gold.ac.uk/~mas03dm/papers/PawleyMullensiefen_Singalong_2012.pdf


                • music / sound as a weapon:
                  https://www.newyorker.com/magazine/2016/07/04/when-music-is-violence
                  The New Yorker - July 4, 2016, issue, with the headline “The Sound of Hate.”


                • see also the role singing played in the World War I Christmas Truce of 1914


                • Stanford University psychologists Scott S. Wiltermuth and Chip Heath conducted a series of experiments to see how synchronous movement affects group interactions.[...]These findings suggest that cultural practices which involve synchrony (such as dancing, singing or marching) may enable groups to produce members who are cooperative and willing to make personal sacrifices, for the benefit of the group.(Journal Reference:
                  Synchrony and Cooperation. Psychological Science, January 2009)


                • there are quite a few videos on youtube showing russian soldiers marching while chanting the theme song to spongebob squarepants (;
                  https://www.youtube.com/watch?v=vhuzb3WMntc





                share|improve this answer











                $endgroup$

















                  3












                  $begingroup$

                  Same old, same old...to boost morale, increase hormone levels and pulse to sharpen senses and raise euphoria / aggression, tighten team bonding and focus (stronger together)



                  I'd love to list some points of the following but it is just too much tl;dr info.



                  • An interesting study about singing along to (in this case western pop) music:
                    http://www.doc.gold.ac.uk/~mas03dm/papers/PawleyMullensiefen_Singalong_2012.pdf


                  • music / sound as a weapon:
                    https://www.newyorker.com/magazine/2016/07/04/when-music-is-violence
                    The New Yorker - July 4, 2016, issue, with the headline “The Sound of Hate.”


                  • see also the role singing played in the World War I Christmas Truce of 1914


                  • Stanford University psychologists Scott S. Wiltermuth and Chip Heath conducted a series of experiments to see how synchronous movement affects group interactions.[...]These findings suggest that cultural practices which involve synchrony (such as dancing, singing or marching) may enable groups to produce members who are cooperative and willing to make personal sacrifices, for the benefit of the group.(Journal Reference:
                    Synchrony and Cooperation. Psychological Science, January 2009)


                  • there are quite a few videos on youtube showing russian soldiers marching while chanting the theme song to spongebob squarepants (;
                    https://www.youtube.com/watch?v=vhuzb3WMntc





                  share|improve this answer











                  $endgroup$















                    3












                    3








                    3





                    $begingroup$

                    Same old, same old...to boost morale, increase hormone levels and pulse to sharpen senses and raise euphoria / aggression, tighten team bonding and focus (stronger together)



                    I'd love to list some points of the following but it is just too much tl;dr info.



                    • An interesting study about singing along to (in this case western pop) music:
                      http://www.doc.gold.ac.uk/~mas03dm/papers/PawleyMullensiefen_Singalong_2012.pdf


                    • music / sound as a weapon:
                      https://www.newyorker.com/magazine/2016/07/04/when-music-is-violence
                      The New Yorker - July 4, 2016, issue, with the headline “The Sound of Hate.”


                    • see also the role singing played in the World War I Christmas Truce of 1914


                    • Stanford University psychologists Scott S. Wiltermuth and Chip Heath conducted a series of experiments to see how synchronous movement affects group interactions.[...]These findings suggest that cultural practices which involve synchrony (such as dancing, singing or marching) may enable groups to produce members who are cooperative and willing to make personal sacrifices, for the benefit of the group.(Journal Reference:
                      Synchrony and Cooperation. Psychological Science, January 2009)


                    • there are quite a few videos on youtube showing russian soldiers marching while chanting the theme song to spongebob squarepants (;
                      https://www.youtube.com/watch?v=vhuzb3WMntc





                    share|improve this answer











                    $endgroup$



                    Same old, same old...to boost morale, increase hormone levels and pulse to sharpen senses and raise euphoria / aggression, tighten team bonding and focus (stronger together)



                    I'd love to list some points of the following but it is just too much tl;dr info.



                    • An interesting study about singing along to (in this case western pop) music:
                      http://www.doc.gold.ac.uk/~mas03dm/papers/PawleyMullensiefen_Singalong_2012.pdf


                    • music / sound as a weapon:
                      https://www.newyorker.com/magazine/2016/07/04/when-music-is-violence
                      The New Yorker - July 4, 2016, issue, with the headline “The Sound of Hate.”


                    • see also the role singing played in the World War I Christmas Truce of 1914


                    • Stanford University psychologists Scott S. Wiltermuth and Chip Heath conducted a series of experiments to see how synchronous movement affects group interactions.[...]These findings suggest that cultural practices which involve synchrony (such as dancing, singing or marching) may enable groups to produce members who are cooperative and willing to make personal sacrifices, for the benefit of the group.(Journal Reference:
                      Synchrony and Cooperation. Psychological Science, January 2009)


                    • there are quite a few videos on youtube showing russian soldiers marching while chanting the theme song to spongebob squarepants (;
                      https://www.youtube.com/watch?v=vhuzb3WMntc






                    share|improve this answer














                    share|improve this answer



                    share|improve this answer








                    edited Aug 28 '18 at 13:58









                    Tim B

                    61.4k24173294




                    61.4k24173294










                    answered Aug 26 '18 at 17:10









                    DigitalBlade969DigitalBlade969

                    3064




                    3064





















                        2












                        $begingroup$


                        Right that moment I was feeling unusually expendable, almost expended, because I was hearing the sweetest sound in the universe, the beacon the retrieval boat would land on, sounding our recall. The beacon is a fast insertion ship, fired ahead of the retrieval boat, containing a single crew member that starts singing that welcome, welcome music. The retrieval boat homes in on it automatically three minutes later and you had better be on hand, because the bus can’t wait and there won’t be another one along.



                        But you don’t walk away on another cap trooper, not while there’s a chance he’s still alive — not in Rasczak’s Roughnecks. Not in any outfit of the Mobile Infantry. You try to make pickup.



                        I heard Jelly order: "Fleads up, lads! Close to retrieval circle and interdict! On the bounce!"



                        And I heard the Battle-singer Cicad’s sweet voice: "— to the everlasting glory of the infantry, shines the name, shines the name of Rodger Young!" and I wanted to head for it so bad I could taste it.




                        Robert A. Heinlein, Starship Troopers, modified.



                        In this modern day and age, Cicad plays a central role in in his platoon, due to his unique skills. Every mission ends with his Retrieval Team serving as a beacon for the landing troops to gather and prepare for extraction, his legendary presence and powerful voice serving as a boost to the Infantry troops.






                        share|improve this answer









                        $endgroup$

















                          2












                          $begingroup$


                          Right that moment I was feeling unusually expendable, almost expended, because I was hearing the sweetest sound in the universe, the beacon the retrieval boat would land on, sounding our recall. The beacon is a fast insertion ship, fired ahead of the retrieval boat, containing a single crew member that starts singing that welcome, welcome music. The retrieval boat homes in on it automatically three minutes later and you had better be on hand, because the bus can’t wait and there won’t be another one along.



                          But you don’t walk away on another cap trooper, not while there’s a chance he’s still alive — not in Rasczak’s Roughnecks. Not in any outfit of the Mobile Infantry. You try to make pickup.



                          I heard Jelly order: "Fleads up, lads! Close to retrieval circle and interdict! On the bounce!"



                          And I heard the Battle-singer Cicad’s sweet voice: "— to the everlasting glory of the infantry, shines the name, shines the name of Rodger Young!" and I wanted to head for it so bad I could taste it.




                          Robert A. Heinlein, Starship Troopers, modified.



                          In this modern day and age, Cicad plays a central role in in his platoon, due to his unique skills. Every mission ends with his Retrieval Team serving as a beacon for the landing troops to gather and prepare for extraction, his legendary presence and powerful voice serving as a boost to the Infantry troops.






                          share|improve this answer









                          $endgroup$















                            2












                            2








                            2





                            $begingroup$


                            Right that moment I was feeling unusually expendable, almost expended, because I was hearing the sweetest sound in the universe, the beacon the retrieval boat would land on, sounding our recall. The beacon is a fast insertion ship, fired ahead of the retrieval boat, containing a single crew member that starts singing that welcome, welcome music. The retrieval boat homes in on it automatically three minutes later and you had better be on hand, because the bus can’t wait and there won’t be another one along.



                            But you don’t walk away on another cap trooper, not while there’s a chance he’s still alive — not in Rasczak’s Roughnecks. Not in any outfit of the Mobile Infantry. You try to make pickup.



                            I heard Jelly order: "Fleads up, lads! Close to retrieval circle and interdict! On the bounce!"



                            And I heard the Battle-singer Cicad’s sweet voice: "— to the everlasting glory of the infantry, shines the name, shines the name of Rodger Young!" and I wanted to head for it so bad I could taste it.




                            Robert A. Heinlein, Starship Troopers, modified.



                            In this modern day and age, Cicad plays a central role in in his platoon, due to his unique skills. Every mission ends with his Retrieval Team serving as a beacon for the landing troops to gather and prepare for extraction, his legendary presence and powerful voice serving as a boost to the Infantry troops.






                            share|improve this answer









                            $endgroup$




                            Right that moment I was feeling unusually expendable, almost expended, because I was hearing the sweetest sound in the universe, the beacon the retrieval boat would land on, sounding our recall. The beacon is a fast insertion ship, fired ahead of the retrieval boat, containing a single crew member that starts singing that welcome, welcome music. The retrieval boat homes in on it automatically three minutes later and you had better be on hand, because the bus can’t wait and there won’t be another one along.



                            But you don’t walk away on another cap trooper, not while there’s a chance he’s still alive — not in Rasczak’s Roughnecks. Not in any outfit of the Mobile Infantry. You try to make pickup.



                            I heard Jelly order: "Fleads up, lads! Close to retrieval circle and interdict! On the bounce!"



                            And I heard the Battle-singer Cicad’s sweet voice: "— to the everlasting glory of the infantry, shines the name, shines the name of Rodger Young!" and I wanted to head for it so bad I could taste it.




                            Robert A. Heinlein, Starship Troopers, modified.



                            In this modern day and age, Cicad plays a central role in in his platoon, due to his unique skills. Every mission ends with his Retrieval Team serving as a beacon for the landing troops to gather and prepare for extraction, his legendary presence and powerful voice serving as a boost to the Infantry troops.







                            share|improve this answer












                            share|improve this answer



                            share|improve this answer










                            answered Aug 28 '18 at 13:39









                            OnoSendaiOnoSendai

                            4,30511023




                            4,30511023





















                                0












                                $begingroup$

                                The "Dynamo" character in Schwarzenegger's "Running Man" was singing before he started zapping people. That's a bit in the future, but opportunities for a murderous singing assassin appearing on TV might arise.






                                share|improve this answer









                                $endgroup$








                                • 1




                                  $begingroup$
                                  Seems to me rather far from the premise of the question. That was an arena being shown for entertainment, and the Runners were meant to be unarmed.
                                  $endgroup$
                                  – can-ned_food
                                  Aug 28 '18 at 5:45















                                0












                                $begingroup$

                                The "Dynamo" character in Schwarzenegger's "Running Man" was singing before he started zapping people. That's a bit in the future, but opportunities for a murderous singing assassin appearing on TV might arise.






                                share|improve this answer









                                $endgroup$








                                • 1




                                  $begingroup$
                                  Seems to me rather far from the premise of the question. That was an arena being shown for entertainment, and the Runners were meant to be unarmed.
                                  $endgroup$
                                  – can-ned_food
                                  Aug 28 '18 at 5:45













                                0












                                0








                                0





                                $begingroup$

                                The "Dynamo" character in Schwarzenegger's "Running Man" was singing before he started zapping people. That's a bit in the future, but opportunities for a murderous singing assassin appearing on TV might arise.






                                share|improve this answer









                                $endgroup$



                                The "Dynamo" character in Schwarzenegger's "Running Man" was singing before he started zapping people. That's a bit in the future, but opportunities for a murderous singing assassin appearing on TV might arise.







                                share|improve this answer












                                share|improve this answer



                                share|improve this answer










                                answered Aug 27 '18 at 16:58









                                gnasher729gnasher729

                                83747




                                83747







                                • 1




                                  $begingroup$
                                  Seems to me rather far from the premise of the question. That was an arena being shown for entertainment, and the Runners were meant to be unarmed.
                                  $endgroup$
                                  – can-ned_food
                                  Aug 28 '18 at 5:45












                                • 1




                                  $begingroup$
                                  Seems to me rather far from the premise of the question. That was an arena being shown for entertainment, and the Runners were meant to be unarmed.
                                  $endgroup$
                                  – can-ned_food
                                  Aug 28 '18 at 5:45







                                1




                                1




                                $begingroup$
                                Seems to me rather far from the premise of the question. That was an arena being shown for entertainment, and the Runners were meant to be unarmed.
                                $endgroup$
                                – can-ned_food
                                Aug 28 '18 at 5:45




                                $begingroup$
                                Seems to me rather far from the premise of the question. That was an arena being shown for entertainment, and the Runners were meant to be unarmed.
                                $endgroup$
                                – can-ned_food
                                Aug 28 '18 at 5:45

















                                draft saved

                                draft discarded
















































                                Thanks for contributing an answer to Worldbuilding Stack Exchange!


                                • Please be sure to answer the question. Provide details and share your research!

                                But avoid


                                • Asking for help, clarification, or responding to other answers.

                                • Making statements based on opinion; back them up with references or personal experience.

                                Use MathJax to format equations. MathJax reference.


                                To learn more, see our tips on writing great answers.




                                draft saved


                                draft discarded














                                StackExchange.ready(
                                function ()
                                StackExchange.openid.initPostLogin('.new-post-login', 'https%3a%2f%2fworldbuilding.stackexchange.com%2fquestions%2f123060%2fin-modern-day-combat-what-would-warrant-singing-during-battle%23new-answer', 'question_page');

                                );

                                Post as a guest















                                Required, but never shown





















































                                Required, but never shown














                                Required, but never shown












                                Required, but never shown







                                Required, but never shown

































                                Required, but never shown














                                Required, but never shown












                                Required, but never shown







                                Required, but never shown







                                Popular posts from this blog

                                𛂒𛀶,𛀽𛀑𛂀𛃧𛂓𛀙𛃆𛃑𛃷𛂟𛁡𛀢𛀟𛁤𛂽𛁕𛁪𛂟𛂯,𛁞𛂧𛀴𛁄𛁠𛁼𛂿𛀤 𛂘,𛁺𛂾𛃭𛃭𛃵𛀺,𛂣𛃍𛂖𛃶 𛀸𛃀𛂖𛁶𛁏𛁚 𛂢𛂞 𛁰𛂆𛀔,𛁸𛀽𛁓𛃋𛂇𛃧𛀧𛃣𛂐𛃇,𛂂𛃻𛃲𛁬𛃞𛀧𛃃𛀅 𛂭𛁠𛁡𛃇𛀷𛃓𛁥,𛁙𛁘𛁞𛃸𛁸𛃣𛁜,𛂛,𛃿,𛁯𛂘𛂌𛃛𛁱𛃌𛂈𛂇 𛁊𛃲,𛀕𛃴𛀜 𛀶𛂆𛀶𛃟𛂉𛀣,𛂐𛁞𛁾 𛁷𛂑𛁳𛂯𛀬𛃅,𛃶𛁼

                                Edmonton

                                Crossroads (UK TV series)